Инфоурок Математика Другие методич. материалыСборник методических указаний для выполнения практических работ дисциплины "Математика: алгебра и начала математического анализа, геометрия"

Сборник методических указаний для выполнения практических работ дисциплины "Математика: алгебра и начала математического анализа, геометрия"

Скачать материал

Сборник методических указаний для проведения практических работ стр. 1 из 121

Министерство образования и науки Самарской областиhello_html_m57aa3027.jpg

Государственное бюджетное профессиональное образовательное

учреждение Самарской области

«Тольяттинский политехнический колледж»

(ГБПОУ СО «ТПК»)


УТВЕРЖДАЮ

Заместитель директора по УР

___________ С.А.Гришина

___ ____________ 2016




СБОРНИК МЕТОДИЧЕСКИХ УКАЗАНИЙ

ДЛЯ ВЫПОЛНЕНИЯ ПРАКТИЧЕСКИХ РАБОТ

ДИСЦИПЛИНЫ МАТЕМАТИКА: АЛГЕБРА И НАЧАЛА МАТЕМАТИЧЕСКОГО АНАЛИЗА; ГЕОМЕТРИЯ


07.02.01 Архитектура

08.02.01 Строительство и эксплуатация зданий

и сооружений

08.02.04 Водоснабжение и водоотведение

08.02.05 Строительство и эксплуатация автомобильных

дорог и аэродромов

09.02.03 Программирование в компьютерных системах

15.02.08 Технология машиностроения

21.02.05 Земельно-имущественные отношения

21.02.06 Информационные системы обеспечения

градостроительной деятельности

22.02.06 Сварочное производство

23.02.03 Техническое обслуживание и ремонт

автомобильного транспорта

38.02.07 Банковское дело

40.02.01 Право и организация социального обеспечения



Тольятти, 2016






ОДОБРЕНА


Протокол ПЦК ЕНД

от ___ _____20__ № ____

Председатель ПЦК ЕНД

________ Л.А. Гончарова

___ ______ 20___










Сборник методических указаний разработан Захаровой С.В., Лабгаевой Э.В.,

Волковой А.В., Мельниковой Е.Ю.– преподавателями дисциплины «Математика: алгебра и начала математического анализа; геометрия» ГБПОУ СО «ТПК»





Сборник методических указаний для выполнения практических работ составлен в соответствии с рабочей программой дисциплины «Математика: алгебра и начала математического анализа; геометрия», для студентов первого курса технических и социально-экономических специальностей.












Содержание








Введение

Сборник методических указаний для выполнения практических работ дисциплины

«Математика: алгебра и начала математического анализа; геометрия» предназначен для студентов первого курса технических и социально-экономических специальностей. Дисциплина «Математика: алгебра и начала математического анализа; геометрия» в соответствии с рабочей программой рассчитана на 234 часа, из них 34 часа отведено на проведение практических занятий. Практические занятия направлены на проверку усвоения и закрепление материала, изученного на теоретических занятиях.

Сборник методических указаний содержит 17 практических работ, в каждой из

которых имеются:

  • краткие теоретические сведения

  • образец решений задач

  • задания для самостоятельного решения

Методическая разработка рекомендуется для использования преподавателями, ведущими данный предмет в средних специальных учебных заведениях.



















Практическая работа №1

Тема: «Нахождение приближенных вычислений»

Цель работы: научиться вычислять значения чисел пользуясь правилами

приближений, определять абсолютную и относительную погрешность вычислений

В результате выполнения практической работы студент должен:

знать:

  • определение приближенного значения числа

  • формулы абсолютной и относительной погрешности вычислений

  • понятие стандартного вида числа, мантиссы и порядка числа

уметь:

  • вычислять приближённое значение числа и выполнять действия над числами, пользуясь калькулятором

  • находить абсолютную и относительную погрешность

  • записывать число в стандартном виде


Краткие теоретические сведения

Число а называется приближенным значением числа х, вычисленным с

точностью до h > 0 , если выполняется неравенство \х - а\< h.

Разность называют абсолютной погрешностью, a h — оценкой погрешности приближенного вычисления.

Отношение абсолютной погрешности к приближенному значению числа, т.е. число называют относительной погрешностью вычислений. Часто относительную погрешность указывают в процентах.

Стандартная запись. Приближенные значения величины часто указывают в так

называемой стандартной записи. Положительные числа в стандартной записи представляют в виде: а • 10*, где число а выбирают так, чтобы оно лежало в промежутке [1; 10), т.е. удовлетворяло неравенствам 1 < а < 10, и записывалось десятичной дробью с несколькими знаками после запятой. Число а в стандартной записи х называют мантиссой числа х, а показатель k — его порядком.

Зачем точные значения величины заменяют ее приближенным значением? Прежде всего потому, что вычислить и записать точное значение величины не удается — всегда при измерении величины можно найти ее значение лишь с некоторой точностью. Кроме того, точная информация бывает излишней - часто достаточно знать лишь порядок числа, степень его близости к другим, более просто записываемым числам.

К приближенным числам относятся:

  • результаты измерений, взвешиваний, некоторые основные константы.

  • проектные данные, нормируемые гостами.

  • математические величины.

  • результаты счета предметов, если при повторении получаются разные ответы.

  • результаты округления чисел и действий над приближенными числами.

Основные требования к вычислениям

  • Вычисления надо вести лишь с той степенью точности, которая необходима для практики в данном конкретном случае.

  • По возможности рационализировать процесс вычислений( таблицы, счет приборы) т.к. в курсовых и дипломных проектах важную роль играет быстрота выполнения действий, экономия в труде и во времени.

  • Уметь делать предварительную прикидку или оценить результат.

Значащими цифрами числа считаются те его цифры, расположение которых

остается неизменным при переносе запятой влево или вправо, или это цифры 1…9, употребленные для записи числа и 0 в следующих случаях: а) если 0 стоит между значащими цифрами, б) если 0 стоит в разряде данной точности в конце числа.

Десятичные знаки - все цифры после запятой.

Практические приемы приближенных вычислений

  • При сложении и вычитании приближенных чисел в результате следует сохранить

столько десятичных знаков, сколько их в приближенном числе с наименьшим числом десятичных знаков, например: 2,31+5,774+27,135,2

  • При умножении и делении в результате сохраняют столько значащих цифр, сколько

их в числе с наименьшем кол-вом значащих цифр, например: 0,24*34,216=8,22248,2

  • При возведении в квадрат (степень) в результате сохраняют столько значащих цифр,

сколько их имеет основание степени, например:

  • При извлечении корня в результате сохраняют столько значащих цифр сколько их и

имеет подкоренное выражение, например:

  • При сложении и вычитании целых чисел их записывают в стандартном виде, вынося

за скобки наибольшую степень и используя правило 1, например:


  • В промежуточных вычислениях, необходимо сохранять на одну цифру больше, чем

это рекомендуется предыдущими правилами. В окончательном результате эта запасная цифра округляется, например:


Образец решения задач

Задание 1. Вычислите с точностью до а: ;,;,

Решение:

в калькуляторе используем кнопку , нажимаем последовательно 12.5 , затем , затем 3.4 = , получаем на экране калькулятора 5364.065153, округляем до , т.е. до сотых, получаем 5364,0651535364,07, т.к. после 6 стоит цифра 5, последнюю цифру на единицу увеличиваем, записываем

, в калькуляторе последовательно используем кнопки и , над которой написано , нажимаем 25.36 и 5 = , получаем на экране калькулятора 1.909105153, округляем до сотых, получаем 1,9091051531,91, записываем

,в калькуляторе используем кнопку, так же на экране калькулятора должна высвечиваться запись , для перехода к ней используется кнопка . Для того, чтобы перевести минуты в градусы учитываем, что . Нажимаем последовательно , получаем на экране калькулятора 0.8935002052, округляем до сотых, получаем 0,89350020520,89, записываем

, в калькуляторе используем кнопку, так же на экране калькулятора должна высвечиваться запись , для перехода к ней используется кнопка , для активизации используют кнопку и кнопку, над которой написано . Нажимаем последовательно , получаем на экране калькулятора -2.414213562, округляем до сотых, получаем -2,414213562-2,41, записываем

Задание 2. Округлите число с точностью до а. Найдите абсолютную и относительную погрешность приближений: ,, ,

Решение:

,. Округляем с точностью до тысячных 12, 556812,557

Найдём абсолютную погрешность приближений

Найдём относительную погрешность , переведём относительную погрешность в проценты, т.е. данное число умножим на 100 (перенесём запятую на 2 единицы вправо), получим

,. Переведём число в десятичную дробь . Округляем с точностью до десятых . Найдём абсолютную погрешность приближений . Найдём относительную погрешность , переведём относительную погрешность в проценты, т.е. данное число умножим на 100 (перенесём запятую на 2 единицы вправо), получим

При решении данной задачи можно опустить период, взяв в качестве точного числа х число 0,222, т.к. для вычислений достаточно взять на одну цифру больше, чем необходимо для расчётов: если мы округляем до десятых, то нужны цифры до сотых, плюс запас на одну больше, т.е. тысячные, остальные цифры не обязательны


Задание 3. Вычислите, пользуясь калькулятором


Решение:

. Нажимаем последовательно на калькуляторе , получаем на экране калькулятора , ответ округляем до десятых, т.к. наименьшее количество знаков после запятой в условии – одна. Получим ответ 24,9

Нажимаем последовательно на калькуляторе, получаем на экране -172.88492

Задание 4. Запишите результат действия в стандартном виде, округлите мантиссу до сотых

Решение:

, здесь 4,86 – мантисса числа, 5 – порядок числа



Задания для самостоятельного решения

Практическая работа №1

Тема: «Нахождение приближенных вычислений»


с точностью до а

Округлите число с точностью до а. Найдите абсолютную и относительную погрешность приближений

Вычислите, пользуясь калькулятором

Запишите результат действия в стандартном виде, округлите мантиссу до сотых

1

;

;



,

,





2

;

;



,

,





3

;

;



,

,




4

;


,

,



hello_html_m1f242778.gif

5

;

;


,

,



6

;

;



,

,



7

;

;



,

,



8

;

;



,

,



9

;

;



,

,



10

;

;



,

,





Практическая работа №2

Тема: «Преобразование алгебраических выражений»

Цель работы: научиться находить значения степени и корня в алгебраических

выражениях с использованием формул степеней и радикалов

В результате выполнения практической работы студент должен:

знать:

  • определение степени и корня

  • формулы для вычисления степеней и радикалов

уметь:

  • преобразовывать алгебраические выражения с использованием формул степеней и

радикалов

Краткие теоретические сведения

Выражение an называется степенью. В этом выражении число a называется

основанием степени, а число n ― показателем степени.

Например, степень 35, имеет основание степени равное 3, а показатель степени равен 5. Это число можно получить, если мы умножим число 3 само на себе 5 раз, т. е. 35 = 3 ∙ 3 ∙ 3 ∙ 3 ∙ 3.

Для положительных чисел a и b и чисел n и m справедливы следующие свойства степени:

1) a0 = 1.

Любое число в нулевой степени равно 1.

2) a1 = a.

Степенью числа с показателем 1 называется само это число.

3) an ∙ am = an+m.

При умножении степеней с одинаковыми основаниями, основание остается прежним, а показатели складываются.

4) hello_html_524655ec.png = an-m.

При делении степеней с одинаковыми основаниями, основание остается прежним, а показатели вычитаются.

5) (ab)n = an ∙ bn.

При возведении в степень произведения, в эту степень возводится каждый множитель.

6) hello_html_2fc82d18.png = hello_html_m326157b2.png .

При возведении в степень дроби в эту степень возводится числитель дроби и знаменатель.

7) (an)m = an∙m.

При возведении степени в степень показатели перемножаются.

8) a-n = hello_html_m59148385.png = hello_html_11a63735.png .

При возведении в отрицательную степень, основание степени «переворачивается», и знак показателя степени меняется на противоположный.

Корнем n-ой степени  n√a  из числа  a  называется число, n-я степень которого

равна a.

Натуральноечисло n называется показателемкорня. Число a называется подкоренным выражением.

Замечание: степень корня ― это натуральное число, большее 1.

Корень второй степени √a называется квадратным корнем и двойка обычно опускается. Корень третьей степени 3√a называется кубическим корнем.

Выделяют корни четной степени (n ― четное) и корни нечетной степени (n ― нечетное):

Корень нечетной степени из положительного числа ― обязательно положительное число.

n√a = b, где ab > 0, n ― нечетное.

Пример: 3√8 = 2, 5√32 = 2, 12√1 = 1.

Корень нечётной степени из отрицательного числа ― отрицательное число, однозначно определенное.

n√a = b, где ab < 0, n ― нечетное.

Пример: 3√–8 = –2, 5√–243 = –3, 9√–1 = –1.

Корень чётной степени берется только из положительного числа.

n√a = b, где ab > 0, n ― четное;

n√a не существует, если а < 0, n ― четное.

Пример: √16 = 4, 4√81 = 3, 14√1 = 1.

Для положительных чисел a и b и чисел n и m справедливы следующие свойства корня:

1) n√an a;

2) n√a ∙ n√b n√ab;

3) hello_html_33e8ce1a.png = hello_html_3a8e1c05.png ;

4) n√am = (n√a)m;

5) hello_html_m35f40f0c.png = nm√a;

6) n√am = am/n.

Для корня четной степени (n ― четное) справедливы ещё ряд свойств (a, b ― любые числа):

1) n√an a;

2) n√ab = hello_html_m3227e14a.png ∙ hello_html_1fe4fe56.png ;

3) hello_html_10f8f26c.png = hello_html_62c0426.png 

Формулы степеней и радикалов представлены в таблице





Образец решения задач

Задание 1. Запишите в виде степени с рациональным показателем

Решение:

Переведём радикалы в степени: , воспользуемся свойствами

умножения и деления чисел с одинаковыми основаниями: , раскроем

скобки и упростим выражение в показателе степени:

Задание 2. Упростите:

Решение:

Задание 3. Запишите как степень 2:

Решение:

Представим число 128 как , а число 8 как, получим:


Задание 4. Упростите: а) , б)

Решение:

а) Преобразуем формулу сокращённого умножения - разность квадратов:

, при возведении степени в степень показатели

перемножаются, поэтому

б) Т.к. при возведении степени в степень показатели перемножаются, получим:

, далее преобразуем разность квадратов


Задание 5. Вычислите:

Решение:

Воспользуемся свойствами степеней:



Задание 6. Сравните: а) hello_html_36cc74a4.gif и ; б) и

Решение:

а) Т.к. имеем одинаковые основания 41>1, сравниваем показатели:

, тогда и hello_html_36cc74a4.gif < ;

б) Здесь и основания и показатели разные, но оба числа положительные, поэтому можно

возвести оба числа в одну и ту же степень, в данном случае в 14-ю, получим

и, получаем, что , действительно 49<2097152, поэтому <

Проверим по калькулятору: 1,32047, 2,82843, 1,32047<2,82843, т.е. <








Задания для самостоятельного решения

Практическая работа № 2.

Тема: «Степень с действительным показателем»



Запишите

как степень 2

Упростите

Вычислите

Сравните

1






а) и ;

б) и

2







а) и ;

б) и

3







а) и ;

б) и

4







а) hello_html_36cc74a4.gif и ;

б) и

5






а) и ;

б) и

6







а) и ;

б) и

7






а) и ;

б) и

8






а) и ;

б) и

9







а) и ;

б) и

10







а) и ;

б) и










Практическая работа №3

«Преобразование логарифмических выражений»

Цель работы: научиться преобразовывать логарифмические выражения

В результате выполнения практической работы студент должен:

знать:

  • определение логарифма числа, основное логарифмическое тождество, свойства логарифмов, формулы перехода

  • определение десятичного и натурального логарифма

  • понятие логарифмирования и потенцирования

уметь:

  • вычислять логарифмы чисел

  • преобразовывать логарифмические выражения

Краткие теоретические сведения

Логарифм положительного числа b по основанию a, где a a это

показатель степени, в которую нужно возвести числа а, чтобы получить b

где

Например: log216 = 4, т.к. 24 = 16


Основное логарифмическое тождество


Свойства логарифмов


Формулы перехода


Десятичный логарифм – логарифм по основанию 10, обозначается , т.е.

Натуральный логарифм логарифм по основанию e, где e – экспонента,, обозначается , т.е.

Логарифмирование - нахождeние логарифма числа по заданному числу

Потенцирование - нахождение числа по заданному логарифму

Образец решения задач

Задание 1. Вычислите значение логарифма: а), б), в)

Решение:

а)

б)

в)

Задание 2 Вычислите значение логарифма: а), б)

Решение:

а)

б)

Задание 3. Вычислите значение числа:

Решение:


Задание 4. Вычислите

Решение:


Задание 5. Вычислите значение числа: а) , б)

Решение:

а)

б)

Задание 6. Вычислите значение числа: а), б)

Решение:

а)

б)

Задание 7. Найдите значение выражения

Решение:


Задание 8. Найдите логарифм при указанном условии

Решение:


, т.к. по условию , получим


Задания для самостоятельного решения

Практическая работа№3

Тема: «Преобразование и вычисление значений логарифмических выражений».


Найти значения логарифмических выражений

Найти


1

2

3

4

5

6

7

8

1









2









3









4









5









6









7









8









9









10











Практическая работа №4

Тема: «Решение показательных и логарифмических уравнений и неравенств»

Цель работы: научиться решать показательные и логарифмические уравнения и

неравенства

В результате выполнения практической работы студент должен:

знать:

  • определение показательного и логарифмического уравнения и неравенства

  • виды показательных и логарифмических уравнений и неравенств и методы их решения

уметь:

  • решать показательные и логарифмические уравнения и неравенства


Краткие теоретические сведения


Показательное уравнение (неравенство) – уравнение (неравенство), в котором

неизвестное находится в показателе степени

Логарифмическое уравнение (неравенство) – уравнение (неравенство), в

котором неизвестное находится под знаком логарифма или в основании логарифма

Основные виды и способы решений показательных и логарифмических уравнений

Способ решения

Вид

Способ решения

Показательные уравнения

Логарифмические уравнения

1 Простейшие уравнения


f(x) =


f(x) =

ОДЗ: f(x)

2 Уравнения, решаемые методом уравнивания


Прологарифмируем

по основанию а

получим

f(x) = g(x)


Пропотенцируем

по основанию а


получим

f(x) = g(x)

ОДЗ:

3 Уравнения, решаемые методов введения новой переменной


Замена:



Замена:

ОДЗ: f(x)>0


Показательные неравенства

Логарифмические неравенства

1 Простейшие неравенства



знак сохраняем


знак меняем


знак сохраняем


знак меняем


2 Неравенства, решаемые методом уравнивания







3 Неравенства, решаемые методов введения новой переменной


Замена:


Замена:



Образец решения задач


Задание 1. Решить показательное уравнение 4x = 8

Решение: 4x = 8 , приводим обе части уравнения к основанию 2

22x = 23 , опускаем основание

2x = 3, выражаем х

x = 3/2

Ответ. x = 3/2

Задание 2. Решить показательное уравнение

Решение: , приводим к основанию 5

Ответ.

Задание 3. Решить показательное уравнение 2x+1 + 4x = 80

Решение: 2x+1 + 4x = 80, используем свойства степеней

2x 21 + 22x = 80

2 2x + (2x)2 – 80 =0

Замена: 2x = t

2t + t2 – 80 = 0

t1 = -10;

t2 = 8;


Возвращаемся к замене

2x = -10 2x = 8;

х1= x2= log28;

x2 =3

Ответ. x2 =3

Задание 4. Решить показательное неравенствоhello_html_75939497.png

Решение:

hello_html_75939497.pngПреобразуем правую часть согласно свойствам степени:

hello_html_m1b321b9c.png

hello_html_786f28e2.png

Основание 2 степени больше единицы, значит, знак неравенства сохраняется:

hello_html_m336450ed.png

hello_html_m447e6619.png

hello_html_263ba2.png

Ответ.


Задание 5 . Решить логарифмическое уравнение

Решение:

Ответ.


Задание 6. Решить логарифмическое уравнение

Решение:

Ответ.

Задание 7. Решить логарифмическое неравенство hello_html_1cadbe7.png

Решение: Уравняем основания логарифмов. Для этого число в правой части представим в виде логарифма с нужным основанием:

hello_html_3ee84a69.png.

Имеем неравенство:

hello_html_m18999bce.png

Основание логарифма меньше единицы, имеем эквивалентную систему:

hello_html_3cbb9135.png

hello_html_31ab09.png

hello_html_m4ccd7feb.png

hello_html_m275c8fed.png


Ответ. hello_html_m275c8fed.png

Задания для самостоятельного решения

Практическая работа №4

Тема: «Решение показательных и логарифмических уравнений и неравенств»


Практическая работа №5


Тема: «Нахождение двугранных углов»


Цель работы: научиться строить линейные углы двугранных углов.


В результате выполнения практической работы студент должен:

знать:

  • определение двугранного угла;

  • определение линейного угла;

уметь:

  • находить линейные углы двугранных углов.


Краткие теоретические сведения











Образец решения задач


Задание 1. Дан многогранник


hello_html_9049d0e.png

Построить линейные углы двугранных углов AD, AB, DC, DC.


Решение:

А) Построение :


hello_html_m6ce46124.png

Доказательство:





B) Построение :

hello_html_m283956d7.png

Доказательство:





C) Построение :



hello_html_m134d5b17.png

Доказательство:




D) Построение :

hello_html_4a565110.png

Доказательство:








Задания для самостоятельного решения

Практическая работа № 5

Тема: «Нахождение двугранных углов»


1

2

3

1

Даны равносторонний треугольник АВС и прямоугольник АВDK. Постройте ЛУДУ при ребре АВ. Докажите, что построенный угол является ЛУДУ АВ.

Основанием пирамиды SABCD является прямоугольник АВСD. SB-высота пирамиды. Постройте ЛУДУ при ребрах DC и AD.

Основанием пирамиды MNPK является прямоугольный треугольник NPK с прямым углом К. Высота пирамиды МО падает в центр описанной окружности. Постройте ЛУДУ при ребрах NK и PK.

2

Даны прямоугольный треугольник АВС с прямым углом С и квадрат АСЕF. Постройте ЛУДУ при ребре АС. Докажите, что построенный угол является ЛУДУ АС.

Основанием пирамиды SАВС является прямоугольный треугольник АВС с прямым углом С. Точка О - середина гипотенузы. SО - высота пирамиды. Построить ЛУДУ при рёбрах АС и ВС.

Основанием пирамиды SABCD является квадрат. Высота пирамиды SO падает в точку пересечения диагоналей. Построить ЛУДУ при ребрах АВ и ВС.

3

Даны прямоугольный равнобедренный треугольник АВС с прямым углом С и квадрат АВЕF. Постройте ЛУДУ при ребре АВ. Докажите, что построенный угол является ЛУДУ АВ.


Основанием пирамиды SАВСD является квадрат ABCD. SD – высота пирамиды. Построить ЛУДУ при рёбрах AB и BC.

Основанием пирамиды SABC является правильный треугольник АВС, О - точка пересечения медиан, SO-высота пирамиды. Постройте ЛУДУ при ребрах АВ, АС, и ВС.

4

Даны равнобедренная трапеция ABCD, у которой AB=CD и прямоугольник ВСЕF. Постройте ЛУДУ при ребре ВС. Докажите, что построенный угол является ЛУДУ ВС.


Основанием пирамиды SABCD является прямоугольник. Точка О принадлежит стороне АВ. SО - высота пирамиды. Построить ЛУДУ при ребрах AD и DС.

Основание пирамиды SABC – треугольник, у которого AB=BC, высотой пирамиды служит ребро SB. Построить ЛУДУ при ребре AC.

5

Даны прямоугольная трапеция ABCD, у которой угол A равен углу В, и равнобедренный треугольник АВЕ, у которого АЕ=ВЕ. Постройте ЛУДУ при ребре АВ. Докажите, что построенный угол является ЛУДУ АВ.

Основанием пирамиды MNPK является прямоуголь-ный треугольник NPK с прямым углом К. МN- высота пирамиды. Постройте ЛУДУ при ребре PK.

Основание пирамиды SABCD-равнобедренная трапеция, у которой АD‌‌//ВС. Точка О- середина ребра АD. SO-высота пирамиды. Построить ЛУДУ при ребре ВС.


6

Даны прямоугольный треугольник ORP с прямым углом О и прямоугольник ORЕF. Постройте ЛУДУ при ребре OR. Докажите, что построенный угол является ЛУДУ OR.



Основанием пирамиды SАВС является равнобедренный прямоугольный треугольник АВС с прямым углом С. SС - высота пирамиды. Построить ЛУДУ при ребре АВ.

Основанием пирамиды SABCD является прямоугольник АВСD. Высота пирамиды SO падает в точку пересечения диагоналей. Постройте ЛУДУ при ребрах BC и CD.

7

Даны равнобедренный треугольник MNС и прямоугольник MNDK. Постройте ЛУДУ при ребре MN. Докажите, что построенный угол является ЛУДУ MN.


Основанием пирамиды SABCD является квадрат SABCD. SА-высота пирамиды. Построить ЛУДУ при ребрах ВC и DС.

Основание пирамиды SABC – прямоугольный треугольник АВС с прямым углом C, SВ – высота пирамиды. Построить ЛУДУ при ребре AC.

8

Даны прямоугольный равнобедренный треугольник MNС с прямым углом С и прямоугольник АВMN. Постройте ЛУДУ при ребре MN. Докажите, что построенный угол является ЛУДУ MN.

Основанием пирамиды SABC является правильный треугольник ABC. SА-высота пирамиды. Постройте ЛУДУ при ребре ВС.

Основание пирамиды SАВСД - квадрат. Высота пирамиды SО падает в центр вписанной в квадрат окружности. Построить ЛУДУ при рёбрах АВ и ВС.

9

Даны прямоугольная трапеция ABCD, у которой угол A равен углу В, и прямоугольник АВЕF. Постройте ЛУДУ при ребре АВ. Докажите, что построенный угол является ЛУДУ АВ.

Основанием пирамиды SABCD является квадрат. Точка О принадлежит стороне ВС. SО - высота пирамиды. Построить ЛУДУ при ребрах AD и DС.

Основание пирамиды SАВС - прямоугольный треугольник АВС с прямым углом С. Высота пирамиды SО падает в центр описанной окружности. Построить ЛУДУ при рёбрах АС и ВС

10

Даны прямоугольная трапеция ABCD, у которой угол A равен углу В, и прямоугольный треугольник АВЕ, у которого угол А-прямой. Постройте ЛУДУ при ребре АВ. Докажите, что построенный угол является ЛУДУ АВ.

Основанием пирамиды SMNK является равнобедренный треугольник MNK, у которого MK=MN. SM-высота пирамиды. Постройте ЛУДУ при ребре KN.

Основанием пирамиды SАВСD является прямоугольник АВСD. SА.- высота пирамиды. Построить ЛУДУ при рёбрах СВ и DС

Практическая работа №6

Тема: «Решение комбинаторных задач»


Цель работы: научиться решать комбинаторные задачи.

В результате выполнения практической работы студент должен:

знать:

  • формулы комбинаторики: перестановки, размещения, сочетания;

уметь:

  • решать комбинаторные задачи.


Краткие теоретические сведения


  1. Перестановки

Пусть имеется n различных объектов. Будем переставлять их всеми возможными способами (число объектов остается неизменными, меняется только их порядок). Получившиеся комбинации называются перестановками, а их число равно

Pn=n!=123...(n−1)n

Символ n! называется факториалом и обозначает произведение всех целых чисел от 1до n.

По определению, считают, что 0!=1,1!=1.

  1. Размещения

Пусть имеется n различных объектов. Будем выбирать из них m объектов и переставлять всеми возможными способами между собой (то есть меняется и состав выбранных объектов, и их порядок).Получившиеся комбинации называются размещениями из n объектов по m, а их число равно

  1. Сочетания

Пусть имеется n различных объектов. Будем выбирать из них m объектов все возможными способами (то есть меняется состав выбранных объектов, но порядок не важен). Получившиеся комбинации называются сочетаниями из n объектов по m, а их число равно




Образец решения задач


  1. Сколько четырёхзначных чисел можно составить из четырёх карточек с цифрами 1, 5, 7, 9?



  1. Сколько трёхзначных чисел можно составить из четырёх карточек с цифрами 1, 5, 7, 9, если цифры не повторяются?



  1. Сколькими способами из колоды в 36 карт можно выбрать 3 карты?



  1. Сколькими способами из колоды в 36 карт можно выбрать 1 даму и 2 туза?



  1. Сколькими способами можно рассадить 6 человек за столом?





  1. Студенческая группа состоит из 23 человек, среди которых 10 юношей и 13 девушек. Сколькими способами можно выбрать 2-х человек одного пола?





  1. Группу из 20 студентов нужно разделить на 3 бригады, причем в первую бригаду должны входить 3 человека, во вторую — 5 и в третью — 12. Сколькими способами это можно сделать.



  1. В шахматном турнире принимали участие 15 шахматистов, причем каждый из них сыграл только одну партию с каждым из остальных. Сколько всего партий было сыграно в этом турнире?



  1. Сколько различных дробей можно составить из чисел 3, 5, 7, 11, 13, 17 так, чтобы в каждую дробь входили 2 различных числа?



  1. Имеется 3 фрукта. Сколькими способами можно взять хотя бы один фрукт?



или находим противоположное событие

Задания для самостоятельного решения

Практическая работа №6

Тема: «Решение комбинаторных задач»

1

2

3

4

5

6

1

В вазе 12 хризантем и 7 роз. Сколькими способами можно сделать букет из 5 хризантем.

Из города А в В можно добраться 4 дорогами, из В в С ведут 2 дороги, из С в Д 3 дороги. Сколькими путями можно добраться из В в Д?

Сколькими способами можно составить список из 10 человек?

У мамы 2 яблока и

3 груши. Каждый день в течение 5 дней подряд она съедает по одному фрукту. Сколькими способами это можно сделать?

Сколько можно составить сигналов из шести флажков различного цвета, взятых по два?

Сколькими способами можно распределить 12 классных комнат по 12 учебных кабинетов?

2

В вазе 7 роз и 6 гвоздик. Сколькими способами можно выбрать 2 розы и 3 гвоздики?

Из города A в B можно добраться 4 способами,

из B и C ведут 2дороги,
из C и D 3 дороги. Сколькими путями можно добраться из A и C.

Тридцать учащихся обменялись друг с другом фотокарточками. Сколько всего было роздано фотокарточек?


В лагере 10 друзей решили по приезду домой написать каждому по письму. Сколько было писем?

В библиотеке имеются книги по 16-ти разделам науки. Поступило четыре заказа на литературу. Сколько существует способов получения заказа?

Сколькими способами можно составить комиссию для приёма экзамена по математике из двух преподавателей, если в колледже

3

В вазе 7 цветов: 3 розы и 4 гвоздики. Сколькими способами можно выбрать 3 цветка?

Из города А в В можно добраться четырьмя дорогами. Из В в С ведут две дороги, из С в D три дороги. Сколькими путями можно добраться из А в D?

Сколькими способами можно расположить

в ряд 5 черных и

5 белых шашек?

В кухне 5 лампочек. Сколько существует способов освещения?

Семь одинаковых шариков рассыпаются по четырём лункам. Сколько существует способов распределения шариков по лункам?

Сколькими способами в группе из двадцати человек можно выбрать старосту и физорга, при условии, что каждый учащийся может быть выбран только на одну из этих должностей?

4

В вазе 7 гвоздик и

2 розы. Сколькими способами можно составить букет, состоящий из 2 роз и 3 гвоздик?

На вершину горы ведут 7 дорог. Сколькими способами турист может подняться на нее и спуститься вниз, если подъём и спуск осуществиться различным путём?

Сколько существует способов рассадить

10 гостей по 10 местам за праздничный стол?

4 учащихся сдают экзамен. Сколькими способами может быть поставлена им отметка, если известно, что никто из них не получил неудовлетворительную оценку?

Сколькими способами может быть составлена комиссия для приёма экзамена по математике из двух преподавателей, если в колледже всего пять учителей математики?

Сколькими способами можно составить список из пяти человек?


5

Сколькими способами можно выделить делегацию из

5 человек, выбирая

из 7 женщин
и 8 мужчин, если в делегацию должны входить 3 женщины

В магазине «Все для чая» есть 5 разных чашек и 3 разных блюдца. Сколькими способами можно купить чашку с блюдцем?

Сколько существует способов рассадить квартет из басни Крылова

На железной станции

4 светофора. Сколько можно дать различных сигналов, если каждый
светофор имеет 3 состояния: красный, желтый, зеленый.

Сколькими способами в группе из двадцати человек можно выбрать старосту и физорга, при условии, что каждый учащийся может быть выбран только на одну из этих должностей?

Сколько можно составить сигналов из шести флажков различного цвета, взятых по два?

6

Сколько сигналов можно подать пятью различными флажками, поднимая их в любом количестве и в произвольном порядке?

Туристическая фирма планирует посещение туристами в Италии трех городов: Венеции, Рима и Флоренции. Сколько существует вариантов такого маршрута?

Из цифр 0,1,2,3,4,5,6,7,8,9 образуются всевозможные двузначные числа. Найдите количество всех этих чисел?

Сколько семизначных чисел можно образовать с помощью семи различных цифр, отличных от 0?

Сколько шестизначных чисел, кратных 5 можно составить из чисел 1, 2, 3, 4, 5, 6, при условии, что в числе цифры не повторяются?

Сколькими способами из девяти рабочих можно выделить четырёх для работы на определённом участке

7

Сколькими способами можно выделить делегацию из 5человек, выбирая из 7 женщин и 8 мужчин, если в делегацию должны входить 5 женщин?

4 учащихся сдают экзамен. Сколькими способами может быть поставлена им отметка, если известно, что никто из них не получил неудовлетворительную оценку?

Сколькими способами можно выбрать в группе из 25 человек старосту, профорга,
физорга на равных условиях.


В библиотеке имеются книги по 16-ти разделам науки. Поступило четыре заказа на литературу. Сколько существует способов получения заказа?


Сколько матчей будет сыграно в футбольном чемпионате с участием 16 команд, если каждые две команды встречаются между собой один раз?

Для дежурства в классе, в течение недели (кроме воскресенья) выделено 6 учащихся. Сколькими способами можно установить очередность дежурств, если каждый учащийся дежурит 1 раз?

8

Сколькими способами можно составить делегацию из 4человек, выбирая из 8 мужчин и 5 женщин, если в делегацию должны входить только мужчины или только женщины?

В правлении фирмы входят 5 человек. Из своего состава правление должно выбрать  президента и вице-президент. Сколькими способами это можно сделать?

Сколькими способами можно выбрать 5человек на 6 должностей из 8 кандидатов на эту должность?

Сколькими способами можно составить делегацию в составе 2 человек, выбирая из 5 супружеских пар, если в делегацию должны входить 1 женщина, и 1мужчина?

Для дежурства в классе, в течение недели (кроме воскресенья) выделено 6 учащихся. Сколькими способами можно установить очередность дежурств, если каждый учащийся дежурит 1 раз?

Сколько матчей будет сыграно в футбольном чемпионате с участием 16 команд, если каждые две команды встречаются между собой один раз?

9

Сколькими способами можно выделить делегацию из 4 человек, выбирая из 7 женщин и 2 мужчин?


Сколько различных музыкальных фраз можно составить из 6 нот, если не допускать в одной фразе повторения звуков?

Сколькими способами из семи предметов можно выбрать три?

Сколькими различными способами можно выбрать из 15 человек делегацию в составе трех человек?

Сколько различных перестановок букв можно сделать в слове «КОЛОКОЛ»

Сколькими способами можно выбрать из множества два элемента?

10

В вазе 12 хризантем и 7 роз. Сколькими способами

можно сделать букет из 5 хризантем.

Сколько существует способов рассадить квартет из басни Крылова

Сколькими способами можно разделить группу в 12 человек так, чтобы в одной группе было 5 человек, а в другой 7?

Семь одинаковых шариков рассыпаются по 4 лункам. Сколько существует способов распределения шариков по лункам?

Сколькими способами из девяти рабочих можно выделить четырёх для работы на определённом участке

30 учащихся обменялись друг с другом фотокарточками. Сколько всего было роздано фотокарточек?




























Практическая работа №7

Тема: «Действия над векторами»


Цель работы: научиться применять вектора при решении геометрических задач.


В результате выполнения расчётно-графической работы студент должен:

знать:

  • определение вектора;

  • операции над векторами;

  • формулы для нахождения координат вектора, модуля вектора, скалярного произведения векторов;

уметь:

  • находить координаты и длины векторов, скалярное произведение векторов, величины углов между векторами.


Краткие теоретические сведения

  1. Определение. Вектор - это направленный отрезок.

  2. Линейные операции над векторами

Сложение векторов

а) геометрически:

Пусть даны два вектора и .

Суммой называют вектор, который идёт из начала вектора в конец вектора при условии, что вектор приложен к концу вектора .





б) аналитически:

Пусть даны два вектора и . Тогда

Вычитание векторов

а) геометрически:

Пусть даны два вектора и .

Разностью называют вектор, который в сумме с вектором даёт вектор.






б) аналитически:

Пусть даны два вектора и . Тогда


Умножение вектора на число

а) геометрически:

Пусть дан вектор и число .

Произведением называется вектор, который коллинеарен вектору , имеет длину и направление такое же, как и вектор , если ,и противоположное, если (рис.3).






рис. 3

б) аналитически:

Пусть дан вектор и число .


Свойства линейных операций

1. (переместительное свойство сложения)

2. (сочетательное свойство сложения)

3. (сочетательное свойство умножения)

4. (распределительное свойство относительно суммы чисел)

5. (распределительное свойство относительно суммы векторов)


Скалярное произведение векторов

Скалярным произведением двух ненулевых векторов и называется число, равное произведению длин этих векторов на косинус угла между ними.

.

Отсюда можно найти величину угла между векторами

Если даны два вектора и , то их скалярное произведение определяется формулой


Свойства скалярного произведения

1. (свойство перестановочности сомножителей)

2. (свойство сочетательности относительно умножения на число)

3. (свойство распределительности суммы векторов)

4.

5.


  1. Нахождение координат вектора, длины вектора

1) Пусть даны две точки , , тогда координаты вектора вычисляются по формуле

2) Пусть даны две точки , , тогда длина вектора вычисляется по формуле

3) Пусть дан вектор , тогда длина вектора вычисляется по формуле


Образец решения задач


Дан треугольник ABC, .

По координатам вершин треугольника ABC средствами векторной алгебры найти:

  1. координаты векторов АВ и АС;

  2. длины векторов АВ и АС;

  3. скалярное произведение векторов АВ и АС;

  4. угол между рёбрами АВ и АС;

  5. площадь треугольника АВС.


Решение:

1)


2)




3)


4)



5)



Ответ: , , , , ,




Задания для самостоятельного решения

Практическая работа №7

Тема: «Действия над векторами»


Даны координаты точек А, В, С.

Построить треугольник АВС.

Найти:

  1. Длины сторон треугольника АВС. Периметр треугольника АВС

  2. Углы треугольника АВС. Сделать проверку

  3. Площадь треугольника АВС


Номер

варианта

Координаты

точки А

Координаты

точки В

Координаты

точки С

1


( -1; 4; 1 )

( 3; 4; -2 )

( 4; 1; -2)

2


( -2; -4; 0 )

( -2; -1; 4 )

( -2; 3; 1 )

3


( 2; -3; 4 )

( 1; 2; -1)

( 3; -2; 1 )

4


( 5; 0; 0 )

( 1; 1; 1 )

( 3; -1; 2 )

5


( 3; -2; 1 )

( 3; 0; 2 )

( 1; 2; 5 )

6


( -1; 4; 1 )

( 3; 4; -2 )

( 5; 2; -1 )

7


( -3; -2; 0 )

( 3; -3; 1 )

( 5; 0; 2 )

8


( -5; -5; 3 )

( -4; 1; 1 )

( 0; -2; 2 )

9


( 0; 4; 6 )

( -3; 5; -1 )

( 1; -2; 3 )

10


( 3; 4; 5 )

( -2; 5; 3 )

( -3; -1; 3 )















Практическая работа №8

Тема: «Преобразования простейших тригонометрических выражений»

Цель работы: закрепить полученные знания, умения и навыки в процессе выполнения упражнений, проверить степень усвоения знаний и сформированности умений

В результате выполнения практической работы студент должен:

знать:

  • Формулы соотношений между тригонометрическими функциями одного аргумента

  • Формулы приведения

  • Формулы сложения двух аргументов

  • Формулы двойного и половинного аргумента

  • Формулы сложения тригонометрических функций

уметь:

  • находить значения тригонометрических выражений на основе определения, используя при необходимости инструментальные средства;

  • пользоваться приближенной оценкой при практических расчетах

  • выполнять преобразования выражений, применяя формулы, связанные со свойствами тригонометрических функций


Краткие теоретические сведения

Выражение, в котором переменная содержится под знаком тригонометрических функций, называют тригонометрическим.

Для преобразования выражений используют свойства тригонометрических функций и формулы тригонометрии.

Знаки тригонометрических функций.

hello_html_m5f3ebba0.gif







Формулы, связывающие тригонометрические функции одного и того же аргумента

hello_html_ma61162e.gif

Формулы сложения двух аргументов

hello_html_m2ef743d5.gif

Формулы двойного и половинного аргумента

hello_html_m60b95127.gif

hello_html_m6d7e4f2a.gif





Формулы сложения тригонометрических функций hello_html_m3fbddba5.gif

Формулы преобразования произведения в суммуhello_html_m598d58db.gif

Формулы приведенияhello_html_5d30d17a.gif

Значение тригонометрических функций некоторых угловhello_html_m4bde847.gif

Образец решения задач


а) Использование основного тригонометрического тождества и его следствия.


Задание 1. Найдите hello_html_190dd1a7.gif, если hello_html_138319e6.gif и hello_html_cede8b0.gif.

Решение:

 hello_html_3331e44f.gifhello_html_m76640c22.gif,

hello_html_m126f17a4.gif

Т.к. hello_html_cede8b0.gif, то hello_html_m59e1b5e5.gif.

Ответ. hello_html_m59e1b5e5.gif

Задание 2. Найдите hello_html_7fd04c56.gif, если hello_html_m7a7eb9f4.gif и hello_html_cede8b0.gif.

Решение:

 hello_html_m397524df.gifhello_html_m6ab699b9.gifhello_html_7697f38b.gif.

Т.к. hello_html_cede8b0.gif, то hello_html_m34397c21.gif.

Ответ. hello_html_m34397c21.gif.

Задание 3. Доказать тождество hello_html_78e2a339.gif.


Решение: Приведем левую часть к 1:hello_html_m20455271.gif
hello_html_180fead3.gif

Тождество доказано.

Задание 4. Доказать тождество: sin4α — cos4α  = sinα  — cos2 α .

Решение: Используя формулу для разности квадратов двух чисел, получаем:

sin4α — cos4α = (sin2α + cos2α) (sin2α — cos2α).

Ho sin2α  + cos2α  = 1.   Поэтому

sin4α — cos4α = sin2α — cos2α, что и требовалось доказать.

Задание 5.    Доказать  тождество: sin4 α + cos4 α — 1 = — 2   sin2α cos2α.

Решение: Покажем, что разность между левой и правой частями. данного тождества  равна  нулю.   

(sin4 α + cos4 α — 1) — (— 2   sin2α cos2α)  = (sin4 α + 2sin2α cos2α + cos4 α) — 1 =

= (sin2α  + cos2α)2 — 1 = 1 — 1 = 0.

Тем самым тождество доказано.

Задание 6. Доказать тождество

hello_html_m3dacae95.gif

Решение: Это тождество   можно   рассматривать   как   пропорцию.   Но чтобы доказать справедливость    пропорции  a/b = c/d, достаточно показать, что произведение ее крайних членов ad равно произведению ее средних членов bc. Так мы поступим и в данном случае. Покажем, что (1 — sin α) (1+ sin α) = cos α • cos α.

Действительно,   (1 — sin α)(1 + sin α) = 1 —sin2α = cos2α.


б) Использование формул двойного угла.


Задание 7. Найдите hello_html_6efd2697.gif, если hello_html_m6170495f.gif.

Решение:

 hello_html_m2e3c52e8.gifhello_html_m23357af9.gifhello_html_6f125d96.gif.

Ответ. hello_html_306a71d9.gif.

Задание 8. Найдите значение выражения hello_html_m314684a9.gif.

Решение:

 hello_html_56c2970f.gif.


в) Формулы приведения

Задание 9. Упростить выражение:

hello_html_m38ead17a.png

Решение:

1) hello_html_m2a613334.png

2) hello_html_41641e8e.png

3) hello_html_m4d1ca96b.png

4) hello_html_3eceb0b2.png

5) hello_html_m2956943a.png

6) hello_html_2c5242c3.png

7) hello_html_m6fea3737.png

hello_html_44bed5f0.png



























Задания для самостоятельного решения

Практическая работа №8

Тема: «Преобразования простейших тригонометрических выражений»



а),

б)


а) sin91350;б)ctg


cos4cos6 – sin1sin3=cos7cos3

2

а),


б)


а) tg13950;б) sin


sin1sin3 - cos4cos6= - cos7cos3

3

а),

б)


а) ctg6300;б)cos


cos6cos4 – sin3sin1=cos3cos7

4

а),

б)


а) сos6750;б) tg


sin3sin1 – cos6cos4= - cos7cos3

5

а),

б)


а) sin4950;б) ctg


cos4cos6 - sin1sin3=cos7cos3

6

а),

б)


а) tg7650;б) sin


cos6cos4 – sin3sin1=cos3cos7

7

а),

б)


а) ctg8100;б) cos


sin1sin3 - cos4cos6= - cos7cos3

8

а),

б)


а) сos9000;б) tg


sin3sin1 – cos6cos4= - cos7cos3

9

а),

б)


а) sin3950;б) ctg


sin5cos7-sin10cos2=

- sin5cos3

10

а),

б)


а) сos5750;б)cos


sin3sin1 – cos6cos4=

- cos7cos3













Практическая работа №9

Тема: «Решение тригонометрических уравнений»

Цель работы: научиться определять вид тригонометрического уравнения и решать тригонометрические уравнения с применением различных методов решения

В результате выполнения практической работы студент должен:

знать:

  • определение тригонометрического уравнения, виды и методы решения тригонометрических уравнений

уметь:

  • решать тригонометрические уравнения различных видов


Краткие теоретические сведения


Простейшие тригонометрические уравнения — это уравнения вида:

cos x = a, sin x = a, tg x = a, ctg x = a


Решить простейшее тригонометрическое уравнение — это значит описать множество значений переменной x, для которых тригонометрическая функция принимает заданное значение a


Простейшие тригонометрические уравнения решают с помощью тригонометрического круга

hello_html_15b1b873.png

hello_html_21bbcbf.jpg


Вид уравнения, его геометрическое представление, общее решение и частные случаи представлены в таблице

hello_html_3c857704.gif

hello_html_1be31edd.gif

hello_html_13410df5.gif

hello_html_m74fbd151.gif

α  [-1;1]

hello_html_m3ea02595.png

X 1 = arcsinα + 2πn, n є Z

X = –arcsinα + π + 2πnn є Z


Общий вид:

hello_html_1e65eca6.gif

hello_html_108cd16c.gif


hello_html_m238ca1a4.gif


hello_html_m454ba6c6.gif

hello_html_45eb1312.gif

α  [-1;1]

hello_html_m371c5182.png

X 1 = arccosa + 2πnn є Z

X 2 = –arccosa + 2πnn є Z


Общий вид:

hello_html_27606554.gif



hello_html_m35a6802b.gif

hello_html_52c4b40a.gif


hello_html_8aceaae.gif

hello_html_7ef7fefd.gif

hello_html_4c8cb10f.gif

hello_html_d2fc78.png

X 1 = arctgα + 2πnn є Z;

X 2= arctgα + π + 2πnn є Z


Общий вид:

hello_html_73d5e675.gif

hello_html_m4c1b814d.gif


hello_html_m238ca1a4.gif



hello_html_m4e5cf2fc.gif

hello_html_4667b743.gif

hello_html_4c8cb10f.gif

hello_html_m5d6c5b9d.png

X 1arcctgα + 2πnn є Z

X 2= arcctgα+π + 2πnn є Z


Общий вид:

hello_html_11c3ac72.gif

hello_html_m4c1b814d.gif

hello_html_52c4b40a.gif

hello_html_m4e5cf2fc.gif


При решении простейшего тригонометрического уравнения можно использовать таблицу значений тригонометрических функций

hello_html_m60e38760.png


Решение любого другого тригонометрического уравнения сводится, как правило, к решению одного или нескольких простейших тригонометрических уравнений


Виды тригонометрических уравнений и методы их решения


  • Тригонометрические уравнения, алгебраические, относительно одной из тригонометрических функций. Это уравнения вида:  

hello_html_2737a696.gifhello_html_5fb0da12.gif 

Данные уравнения приводятся к линейным или квадратным уравнениям относительно одной из тригонометрических функций. Они сводятся к простейшим тригонометрическим уравнениям с помощью замены hello_html_m3ccff21f.gif или hello_html_7a4b0fd1.gif.

Например: , замена sin x = t 2t2 + 3t – 2 = 0


  • Уравнения, решаемые с помощью основных тригонометрических формул, представленных в таблице

Тригонометрические тождества


Формулы двойного угла


Формулы половинного угла


Формулы сложения


Выражение тригонометрических функций через тангенс половинного угла


Преобразование суммы тригонометрических функций в произведение


Преобразование произведения тригонометрических функций в сумму



Здесь используют следующие методы приведения разных функций к одной с помощью тригонометрических формул:

  • Уравнения hello_html_m67bc1ac3.gif не являются с виду

алгебраическими, но их можно свести к алгебраическим, используя основное тригонометрическое тождество. Например: , представить sin2x = 1 – cos2x, или , представить и решить дробно-рациональное уравнение


  • Уравнения вида hello_html_m29506290.gif можно решать при помощи универсальной

тригонометрической подстановки hello_html_47b510e9.gif, воспользовавшись формулами, выражающими hello_html_267bdbb2.gif и hello_html_47de0dca.gif через тангенс половинного угла: hello_html_m7ae745f7.gif и hello_html_4b72b495.gif


Таже такие уравнения решают введением вспомогательного угла: hello_html_m5d5ddd4b.gif. Здесь

используют эквивалентные преобразования: hello_html_2d7a9828.gifВведем обозначения: hello_html_126fe2dd.gif и hello_html_m4a2877e3.gif. Заметим, что выражение в скобках в этом случае преобразуется в косинус разности аргументов:

hello_html_305eb87d.gif.

Таким образом, исходное уравнение приводится к эквивалентному простейшему тригонометри-ческому уравнению: hello_html_m13d10f1f.gif, или hello_html_m55243280.gif, решение которого

hello_html_5a1b78e1.gifhello_html_m361bbd8a.gifhello_html_1048aeb4.gif, hello_html_m59606e66.gif.


  • Разложение на множители. Под разложением на множители понимается представление

данного выражения в виде произведения нескольких множителей. Если в одной части уравнения стоит несколько множителей, а в другой – 0, то каждый множитель приравнивается к нулю. Таким образом, данный множитель можно представить в виде совокупности более простых уравнений. При решении уравнений этим методом нужно пользоваться известными способами разложения на множители алгебраических выражений и использовать различные тригонометрические формулы. Например: представить

  • Уравнения, решаемые понижением порядка. Например: , представить


  • Однородные уравнения – уравнения, в которых каждое слагаемое имеет одну и ту же степень.

Однородные уравнения решаются с помощью деления каждого слагаемого на старшую степень тригонометрических функций (sin или cos).

Однородные тригонометрические уравнения:

hello_html_m25afffb2.gif

hello_html_m76cfeed5.gif                                   ,                                     

hello_html_m60ee0a8b.gif,

hello_html_m5bb2ff41.gif

 называются однородными относительно hello_html_48e3c444.gif и hello_html_43a82834.gif. Они обладают тем свойством, что сумма показаний степеней при hello_html_48e3c444.gif и hello_html_43a82834.gif у всех членов уравнения одинакова. Делением на hello_html_2fb02a9f.gif соответственно уравнения приводятся к алгебраическим уравнениям относительно hello_html_m6fdebd93.gif. При этом, конечно, предполагается, что коэффициент hello_html_m271541.gif. В результате получаем равносильное уравнение, так как разделили на hello_html_m7c4fe967.gif (если бы hello_html_m4a43b915.gif, то из исходного уравнения следует, что и hello_html_m3b5e5d18.gif, а это невозможно, так как hello_html_48e3c444.gif и hello_html_43a82834.gif при одном и том же значении х в нуль не обращаются, ибо всегда hello_html_4fd3414.gif).

Уравнение hello_html_1e3a4c46.gif легко сводится к однородному, если правую часть представить в видеhello_html_m703f0019.gif. После очевидных преобразований получаем hello_html_m197063d7.gif.

Разделив, например, уравнение hello_html_7844b25f.gif на hello_html_m57f56bdd.gif, получим уравнение: hello_html_67d769c1.gif. При hello_html_m77a62dc7.gif эти уравнения эквивалентны, так как если hello_html_10bea23f.gif, то из первого уравнения получим, что и hello_html_m48a27c93.gif, что невозможно (hello_html_m75ee7bc7.gifи hello_html_47de0dca.gif при одном и том же аргументе в нуль не обращаются). Далее из эквивалентного уравнения находим hello_html_7a68fb11.gif, решая квадратное уравнение относительно hello_html_7a68fb11.gif, а по значениям hello_html_7a68fb11.gif - соответствующие значения hello_html_68fe9aca.gif


Образец решения задач

Задание 1. Решить уравнение hello_html_6bc37d7d.png

Решение:

Имеем вертикальную пару точек с абсциссой  hello_html_m30cb901a.png:

hello_html_795549c6.png

Все углы, соответствующие верхней точке, описываются формулой hello_html_703f9d01.png

Аналогично, все углы, соответствующие нижней точке, описываются формулой: hello_html_36cadee7.png. Обе серии решений можно описать одной формулой:

х hello_html_m60dc9a3d.png

Задание 2. Решить уравнение hello_html_447d4c.gif

Решение:

hello_html_447d4c.gif

hello_html_50ae8a95.gif

hello_html_m168611c4.gif

hello_html_77344865.gif

Задание 3. Решить уравнение

hello_html_m2c189cc8.png

Решение:

hello_html_m2db2235d.png

hello_html_6b1798dc.png

hello_html_6b1798dc.png

hello_html_6b1798dc.png

hello_html_6b1798dc.png

Задание 4. Решить уравнение hello_html_m5f3e8619.gif.

Решение:

 Это уравнение является квадратным относительно hello_html_43a82834.gif. Поэтому сделаем замену hello_html_7a4b0fd1.gif. В результате получим уравнение  hello_html_m77850fcb.gif. Его корни:  hello_html_m7b3b48ae.gif ,

то есть получаем уравнение  hello_html_m88425ae.gif или hello_html_30f782c3.gif. Первое уравнение дает hello_html_m3c8434a5.gif.

Второе уравнение не имеет корней.

Ответhello_html_m3c8434a5.gif.

Задание 5. Решить уравнение hello_html_m1ed573c3.gif.

Решение: 

Так как hello_html_28ffab9e.gif, то уравнение можно представить в виде

 hello_html_5708a01.gif

hello_html_2142f76d.gif.

Сделаем замену hello_html_m79b1c452.gif. Получим квадратное уравнение

 hello_html_m7637390c.gif, решая которое, имеем:

  hello_html_m10d2d716.gif,то есть

 hello_html_4881af30.gif. Таким образом, получим два простейших уравнения

 hello_html_2ae746ce.gif или hello_html_6185406e.gif.

Решая их, имеем 

hello_html_45d0154e.gif ,   hello_html_68296571.gif.

Задание 6. Решить уравнение hello_html_m2adc41a8.gif.

Решение:

 Это уравнение является однородным относительно hello_html_48e3c444.gif и hello_html_43a82834.gifПоэтому, разделив его на hello_html_m7ace989b.gif hello_html_70ff8110.gif, получим hello_html_12947976.gif.

Введем новую переменную hello_html_57416399.gif и решим квадратное уравнение

 hello_html_66493ac1.gif.Его корни hello_html_41fcdf6b.gifПолучили два простейших тригонометрических уравнения hello_html_2125f814.gif. Решая их, найдем: 

hello_html_m6792a0fe.gif или hello_html_m1a06d58f.gif.

Задание 7 . Решить уравнение hello_html_m64806c51.gif.

Решение:

 Это уравнение, сводящееся к однородному. Имеем

hello_html_m79c3427d.gif

hello_html_m79c3427d.gif

то есть получили однородное уравнение. Разделив обе части уравнения на hello_html_m7ace989b.gif, получим  hello_html_m1ba52c24.gif. Решая это уравнение, квадратное относительно hello_html_m6fdebd93.gif, найдем, что

 hello_html_m5cfce268.gif либо hello_html_a18a724.gif. Таким образом, hello_html_m6792a0fe.gif или hello_html_mf3b77a0.gif.


Задание 8. Решите уравнение 2 sin x+ 3 cos x = 0.

Решение:

Имеем однородное уравнение первой степени

2 sin x+ 3 cos x = 0 | : cos x ≠ 0,

2 tg x + 3 =0,

tg x = -1,5.

х= arctg (-1,5) + πk, k Z или

х = - arctg 1,5 + πk, k Z


Задание 9. Решить уравнение hello_html_1fa52acf.gif.

Решение:

Используем формулы преобразования произведения тригонометрических функций в сумму, получим уравнение:

hello_html_m1a55daae.gifили hello_html_154b2000.gif.

Разность косинусов преобразуем в произведение

hello_html_m14a85d7.gif, которое равносильно совокупности уравнений:

hello_html_73ea5296.gif hello_html_3cc2f277.gif hello_html_5749df3a.gif

Задание 10 .Решить уравнение hello_html_158dc497.gif

Решение:


Первый способ.

Заменив hello_html_267bdbb2.gif и hello_html_47de0dca.gif через hello_html_m4b829c9c.gif, получим:

hello_html_57396896.gifhello_html_m4a8892e0.gif.

Введем новую переменную: hello_html_47b510e9.gif и получим эквивалентное квадратное уравнение относительно hello_html_m1efe09b.gif: hello_html_24274360.gif,у которого дискриминант равен нулю и, следовательно, имеем единственный корень hello_html_m53ca7ff9.gif.

Задача свелась к решению уравнения:hello_html_77e035a1.gif; hello_html_222118f6.gif;hello_html_3aaaa67b.gif, hello_html_m59606e66.gif.

Второй способ.

Введем вспомогательный угол: hello_html_556ac4c5.gif.

Тогда решение исходного уравнения сразу запишется в виде:

hello_html_5a1b78e1.gifhello_html_m446152eb.gifhello_html_5456662e.gif=hello_html_m446152eb.gifhello_html_4c776043.gif, hello_html_m59606e66.gif. Получили тот же ответ

Задание 11. Решить уравнение hello_html_58dd6e7.gif.

Решение:

Применяя формулу синуса двойного углаhello_html_32fe6f46.gif, получим, hello_html_783a6f14.gif. Полученное уравнение равносильно совокупности уравнений:

hello_html_m6df478be.gif.Решение 1-го уравнения: hello_html_m1f3e5e25.gif.

Уравнение hello_html_5c8b92cd.gif преобразуем к виду hello_html_2ae746ce.gif, имеющему решение 

hello_html_m51126bf7.gif.

Задание 12. Решить уравнение hello_html_35d17021.gif.

Решение:

 Перенесем все члены уравнения в левую часть и разложим ее на множители:

hello_html_6f55f4a5.gif

Отсюда следует, что hello_html_3d805024.gif или hello_html_m3cacc9fa.gif, то есть имеем уравнение hello_html_m5f49d917.gif или hello_html_m88425ae.gif. Решая их, получим hello_html_42d19cfd.gif или hello_html_648c19a8.gif.

Задание 13. Решить уравнение hello_html_m7964794d.gif.

Решение:

Преобразуем по выше приведенным формулам левую и правую части уравнения. В результате получим:

hello_html_m43492aab.gif, иначеhello_html_m790d6ca9.gif, то есть  hello_html_m51bf05e7.gif.

Преобразовывая теперь в произведение сумму косинусов, будем иметь

 hello_html_m51c36f79.gif, откуда hello_html_17e38df2.gif или hello_html_m26d1a0d6.gif.

Задание 14. Решить уравнение: hello_html_2a176bd4.gif

Решение:

Заменяя hello_html_m29db41a2.gif и hello_html_m8243eea.gif, получим однородное уравнение: hello_html_m79fd6284.gif, или hello_html_4b6b6140.gif

Деля на hello_html_m57f56bdd.gif (hello_html_48523e4d.gif), получим: hello_html_m33d6a8d0.gif.

Вводим новую переменную hello_html_7155a36b.gif и получаем квадратное уравнение относительно нее:

hello_html_m69fc48b.gif.Корни этого уравнения: hello_html_747257f7.gif. Далее получаем равносильную совокупность уравнений:

hello_html_m2ee2d91f.gif hello_html_73acde7c.gif





















Задания для самостоятельного решения

Практическая работа №9

Тема: «Решение тригонометрических уравнений»



Практическая работа №10

«Преобразования графиков функций»


Цель работы: научиться строить преобразования сдвига и деформации графиков

простейших функций

В результате выполнения практической работы студент должен:

знать:

  • основные понятия преобразования графиков функций

  • правила простейших преобразований графиков функции

уметь:

  • строить преобразования графиков функций: деформацию и сдвиг


Краткие теоретические сведения


Преобразования графиков функций — это  линейные преобразования функции  y = f(x) или её аргумента x к виду  y = af(kx + b) + m, а  также преобразование  с использованием модуля.

Правила преобразований графиков

оси Ох

График функции y = f (x + a) получается с помощью параллельного переноса графика функции y = f (x) вдоль

оси Ох на а единиц в направлении, противоположном знаку числа а


если « + а » - влево

если « - а » - вправо

hello_html_m1e9c802a.png

y = f (x) + b

Параллельный перенос вдоль

оси Оу

График функции y = f (x) + b получается с помощью параллельного переноса графика функции y = f (x) вдоль оси Оy на b единиц в направлении, имеющем знак числа b


если « + b » - вверх

если « - b » - вниз

hello_html_m1e9c802a.png

y = - f (x)

Симметрия относительно

оси Ох

График функции y = - f (x) получается с помощью симметрии графика функции y = f (x) относительно оси Ох

hello_html_m1e9c802a.png

y = f (- x)

Симметрия относительно

оси Оу

График функции y = f (-x) получается с помощью симметрии графика функции y = f (x) относительно оси Оу

hello_html_56efff00.png

y = fx)

Сжатие (растяжение)

вдоль оси Ох

График функции y = fx) получается с помощью сжатия графика функции

y = f (x) в а раз вдоль оси Ох


при а > 1 — сжатие графика к оси ординат в а раз,

при 0 < а < 1 — растяжение графика от оси ординат в а раз

hello_html_m1e9c802a.png

y = b f (x)

Растяжение (сжатие)

вдоль оси Оу

График функции y = b f (x) получается с помощью растяжения графика функции y = f (x) в b раз вдоль оси Оу


при b > 1 — растяжение графика от оси абсцисс в b раз,

при 0 < b < 1 — сжатие графика к оси абсцисс в b раз

hello_html_m1e9c802a.png


Преобразования

с модулем

Для построения графика функции нужно построить график функции y = f (x) для х > 0, а затем отобразить построенный график симметрично относительно оси Оу


при — график остаётся без изменений,

при x < 0 — график симметрично отражается относительно оси ординат

hello_html_me2a6e8f.png


Преобразования

с модулем

Для построения графика функции нужно построить график функции y = f (x) и отобразить относительно оси Ох те части графика, которые расположены ниже этой оси


при f(x) > 0 — график остаётся без изменений,

при f(x) < 0 — график симметрично отражается относительно оси абсцисс

hello_html_m1e9c802a.png






Образец решения задач

Задание 1 Дан график функции y = f (x)







Постройте графики следующих функций:

  1. y = f (x+1),

  2. y = f (x) +1,

  3. y = f (x-2),

  4. y = f (x) – 2,

  5. y = - f (x),

  6. y = f (-x),

  7. y = 2f (x),

  8. y = f (2x),

  9. y = | g (x) |,

  10. y = g ( |x| )


Решение:


y = f (x-2)


y = f (x) + 2

y = f (x+1)

y = y = f (x) – 1





y = - f (x)





y = f (-x)

y = 2f (x)

y = f (2x)

y = | f (x) |

y = f ( |x| )














Задания для самостоятельного решения

Практическая работа №10

«Преобразования графиков функций»

1

y


x

  1. y = f (x+1)

  2. y = f (x) +1

  3. y = f (2x)

  4. y = 2f (x)

  1. y = f (x/3)

  2. y = 1/2f (x)

  3. y = f (x-2)

  4. y = f (x)-3

  1. y = -f (x)

  2. y = f (-x)

  3. y = 1-f (x)

  4. y = | f (x) |

  5. y = f ( |x| )

2

y


x

  1. y = f (x)-2

  2. y = f (x/2)

  3. y = 1/3f (x)

  4. y = f (x-1)


  1. y = 3f (x)

  2. y = f (x+2)

  3. y = f (x) +3

  4. y = f (3x)

  1. y = -f (x)

  2. y = f (-x)

  3. y = 2-f (x)

  4. y = | f (x) |

  5. y = f ( |x| )

3

y


x


  1. y = f (x+3)

  2. y = f (x) +2

  3. y = f (x)-1

  4. y = f (x-2)


  1. y = 2f (x)

  2. y = f (2x)

  3. y = f (x/3)

  4. y = 1/2f (x)

  1. y = -f (x)

  2. y = f (-x)

  3. y = 3-f (x)

  4. y = | f (x) |

  5. y = f ( |x| )

4

y


x

  1. y = f (x)-3

  2. y = f (x/2)

  3. y = 1/3f (x)

  4. y = f (x-2)


  1. y = 3f (x)

  2. y = f (x+3)

  3. y = f (x) +1

  4. y = f (3x)


  1. y = -f (x)

  2. y = f (-x)

  3. y = 4-f (x)

  4. y = | f (x) |

  5. y = f ( |x| )

5

y


x


  1. y = f (x+4)

  2. y = f (x) +4

  3. y = f (x)-2

  4. y = f (x-2)


  1. y = 2f (x)

  2. y = f (2x)

  3. y = f (x/3)

  4. y = 1/2f (x)

  1. y = -f (x)

  2. y = f (-x)

  3. y = 1-f (x)

  4. y = | f (x) |

  5. y = f ( |x| )



6

y


x

  1. y = f (x)-1

  2. y = f (x/2)

  3. y = 1/3f (x)

  4. y = f (x-3)

  1. y = 3f (x)

  2. y = f (x+1)

  3. y = f (x)+3

  4. y = f (3x)


  1. y = -f (x)

  2. y = f (-x)

  3. y = 2-f (x)

  4. y = | f (x) |

  5. y = f ( |x| )



7

y


x

  1. y = f (x+1)

  2. y = f (x) +2

  3. y = f (x)-3

  4. y = f (x-4)


  1. y = 2f (x)

  2. y = f (2x)

  3. y = f (x/3)

  4. y = 1/2f (x)


  1. y = -f (x)

  2. y = f (-x)

  3. y = 3-f (x)

  4. y = | f (x) |

  5. y = f ( |x| )



8

y


x

  1. y = f (x)-2

  2. y = f (x/2)

  3. y = 1/3f (x)

  4. y = f (x-5)

  1. y = 3f (x)

  2. y = f (x+2)

  3. y = f (x)+2

  4. y = f (3x)


  1. y = -f (x)

  2. y = f (-x)

  3. y = 4-f (x)

  4. y = | f (x) |

  5. y = f ( |x| )



9

y


x

  1. y = f (x+5)

  2. y = f (x) +1

  3. y = f (x)-2

  4. y = f (x-1)


  1. y = 3f (x)

  2. y = f (2x)

  3. y = f (x/3)

  4. y = 1/2f (x)


  1. y = -f (x)

  2. y = f (-x)

  3. y = 1-f (x)

  4. y = | f (x) |

  5. y = f ( |x| )



10

y


x

  1. y = f (x)-3

  2. y = f (x/2)

  3. y = 1/2f (x)

  4. y = f (x-4)

  1. y = 3f (x)

  2. y = f (x+4)

  3. y = f (x)+2

  4. y = f (2x)


  1. y = -f (x)

  2. y = f (-x)

  3. y = 2-f (x)

  4. y = | f (x) |

  5. y = f ( |x| )








Практическая работа №11

Тема: «Графическое решение неравенств»


Цель работы: закрепить полученные знания, умения и навыки в процессе выполнения упражнений, проверить степень усвоения знаний и сформированности умений, научиться решать неравенства графически.


В результате выполнения практической работы студент должен:

знать:

  • определения квадратичной, дробно-линейной, показательной и логарифмической функций;

  • метод интервалов

уметь:

  • строить графики функций (квадратичную, дробно-линейную, показательную, логарифмическую);

  • решать неравенства графически и аналитически.


Краткие теоретические сведения


  1. Квадратичная функция.


Функция вида , где числа и , называется квадратичной функцией.


Графиком квадратичной функции является квадратичная парабола. Если , то ветви параболы направлены вверх, если , то ветви параболы направлены вниз.

Парабола имеет вид:

hello_html_3a3687b9.jpg

Выделенные точки - это, так называемые "базовые точки". Чтобы найти координаты этих точек для функции , составим таблицу:

График  функции  имеет вид:

hello_html_m6d1cabf2.jpg

Для нахождения координат базовых точек составим таблицу:


Следующий важный параметр графика квадратичной функции - координаты вершины параболы: .. Чтобы найти , нужно значение подставить в уравнение параболы или найти значение по формуле:

hello_html_18e68c38.jpg



  1. Дробно-линейная функция.

                                                                                        ax + b
Дробно-линейная функция – это функция вида y = ——— ,
                                                                                         
cx + d

где x – переменная, a, b, c, d – некоторые числа, причем c ≠ 0, ad – bc ≠ 0.

 

hello_html_692dadea.png 

Графиком дробно-линейной функции является гипербола, которую можно получить из гиперболы с помощью параллельных переносов вдоль координатных осей. Для этого формулу дробно-линейной функции надо представить в следующем виде: , где n – количество единиц, на которое гипербола смещается вправо или влево,  m – количество единиц, на которое гипербола смещается вверх или вниз. При этом асимптоты гиперболы сдвигаются в прямые x = m, y = n.

Асимптота – это прямая, к которой приближаются точки кривой по мере их удаления в бесконечность (см.рисунок).

График гиперболы имеет вид.

hello_html_50d82064.png



Для нахождения базовых точек составим таблицу:

-1

-2

-0,5

y

1

0,5

2

-1

-0,5

-2



  1. Показательная функция

Функцию вида y=ax, где а>0, a≠1, х – любое число, называют показательной функцией.

Область определения показательной функции: D (y)=R – множество всех действительных чисел.

Область значений показательной функции: E (y)=R+ - множество всех положительных чисел.

Показательная функция  y=ax возрастает при a>1.

Показательная функция y=ax убывает при 0.

Построим график функции y=2xНайдем значения функции

hello_html_74db9834.jpgпри х=0, х=±1, х=±2, х=±3.

x=0, y=20=1;                   Точка А.

x=1, y=21=2;                   Точка В.

x=2, y=22=4;                   Точка С.

x=3, y=23=8;                   Точка D.              

x=-1, y=2-1=1/2=0,5;       Точка K.

x=-2, y=2-2=1/4=0,25;     Точка M.

x=-3, y=2-3=1/8=0,125;   Точка N.

Большему  значению аргумента х соответствует и большее значение функции у. Функция y=2x возрастает на всей области определения D (y)=R, так как основание функции 2>1.

Построим график функции y=(1/2)x. Найдем значения функции

при х=0, х=±1, х=±2, х=±3.

hello_html_m393f85c9.jpg

x=0, y=(½)0=1;                  Точка A.

x=1, y=(½)1=½=0,5;          Точка B.

x=2, y=(½)2=¼=0,25;        Точка C.

x=3, y=(½)3=1/8=0,125;    Точка D.

x=-1, y=(½)-1=21=2;          Точка K.

x=-2, y=(½)-2=22=4;          Точка M.

x=-3, y=(½)-3=23=8;          Точка N.

  1. Логарифмическая функция

Функцию вида , где a любое положительное число не равное единице, называют логарифмической функцией с основанием а.

Областью определения логарифмической функции будет являться все множество положительных вещественных чисел. Для краткости его еще обозначают R+.

Областью значения логарифмической функции будет являться все множество вещественных чисел.

Логарифмическая функция возрастает при a>1.

Логарифмическая функция убывает при 0.

График логарифмической функции всегда проходит через точку (1;0).

hello_html_7eddc2f5.jpg

hello_html_m643c603b.jpg


Образец решения задач


Задание 1. Решить графически неравенство: 3x>4-x.

Решение:

В одной координатной плоскости построим графики функций: у=3х и у=4-х.

Графики пересеклись в точке А(1; 3). По графику видно, что кривая выше прямой на промежутке .

hello_html_m5be961b0.png

Ответ: .

Задание 2. Решить неравенство графически и аналитически:

Решение:

А) Графически.

В одной координатной плоскости построим графики функций: и .

hello_html_356894a8.png



Найдём точки пересечения графиков:



,

По графику видно, что парабола ниже прямой на отрезке.

Ответ:

B) Аналитически.



Решим неравенство методом интервалов.

Приравняем данное неравенство к нулю:

.

Найдём корни уравнения:

, .

Нанесём найденные значения на числовую ось и определим знак квадратного трёхчлена на каждом числовом промежутке:

hello_html_24e5a63a.png


Ответ:


Задание 3. Решить неравенство графически и аналитически.



Решение:

А) Графически:

В одной координатной плоскости построим графики функций: и .





hello_html_m57246147.png


Найдём точки пересечения графиков:


,

По графику видно, что гипербола выше прямой на интервале.

Ответ:



B) Аналитически.






Решим неравенство методом интервалов.

Приравняем данное неравенство к нулю:


Найдём корни уравнения:




,


Нанесём найденные значения на числовую ось и определим знак квадратного трёхчлена на каждом числовом промежутке:

hello_html_632dbed1.png

Ответ:

















Задания для самостоятельного решения

Практическая работа №11

Тема: «Графическое решение неравенств»


Номер

варианта

1. Решить неравенство графически


2. Решить неравенство графически и аналитически

3. Решить неравенство графически и аналитически


1






2








3






4








5







6








7






8








9






10















Практическая работа №12

Тема: «Построение сечений многогранников»

Цель работы: формирование у учащихся умений и навыков решения задач на построение сечений многогранников, развитие у учащихся пространственного воображения, графической культуры и математической речи.

В результате выполнения практической работы студент должен:

знать:

  • что значит построить сечение многогранника плоскостью;

  • как могут располагаться относительно друг друга многогранник и плоскость;

  • как задается плоскость;

  • пересечение прямой с плоскостью;

  • пересечение плоскостей;

  • свойства параллельных плоскостей;

  • когда задача на построение сечения многогранника плоскостью считается решенной

уметь:

  • строить сечение параллелепипеда плоскостью проходящей через три точки;

  • строить сечение пирамиды плоскостью проходящей через три точки;


Краткие теоретические сведения

Сечение – это изображение фигуры, которая получается при мысленном

рассечении тела плоскостью.

В тетраэдре сечениями могут быть только треугольники или четырехугольники, а в параллелепипеде – треугольники, четырехугольники, пятиугольники или шестиугольники.

Наибольшее число сторон многоугольника, полученного в сечении многогранника плоскостью, равно числу граней многогранника

Правила построения сечений многогранников:

1) проводим прямые через точки, лежащие в одной плоскости;

2) ищем прямые пересечения плоскости сечения с гранями многогранника, для этого

а) ищем точки пересечения прямой принадлежащей плоскости сечения с прямой, принадлежащей одной из граней (лежащие в одной плоскости);

б) параллельные грани плоскость сечения пересекает по параллельным прямым.

Метод следов включает три важных пункта:

Следом называют прямую пересечения плоскости сечения и плоскости какой-либо грани многогранника. Чтобы построить след, достаточно знать две его точки, т. е. точки, лежащие одновременно в секущей плоскости и плоскости рассматриваемой грани. 



Основные правила построения сечений методом следа:

  • Если даны (или уже построены) две точки плоскости сечения на одной грани многогранника, то след сечения этой плоскости – прямая, проходящая через эти три точки.

  • Если дана (или уже построена) прямая пересечения плоскости сечения с основанием многогранника (след на основании) и есть точка, принадлежащая определенной боковой грани, то нужно определить точку пересечения данного следа с этой боковой гранью ( точка пересечения данного следа с общей прямой основания и данной боковой грани)

  • Точку пересечения плоскости сечения с основанием можно определить как точку пересечения какой-либо прямой в плоскости сечения с ее проекцией на плоскость основания.

То есть, суть метода заключается в построении вспомогательной прямой, являющейся изображением линии пересечения секущей плоскости с плоскостью какой-либо грани фигуры. Удобнее всего строить изображение линии пересечения секущей плоскости с плоскостью нижнего основания. Используя след, легко построить изображения точек секущей плоскости, находящихся на боковых ребрах или гранях фигуры. 



Образец решения задач



Задание1. Построить сечение куба, проходящее через точки М, N, L.

hello_html_727bcb87.gif

Решение:

1)

hello_html_m4b6fd7aa.gifhello_html_5bd1859b.gif



2)

hello_html_1e256309.gifhello_html_m5079114d.gif

3)

hello_html_4cf4ede3.gifhello_html_m4c19a477.gif

4)

hello_html_4e62f1eb.gifhello_html_46b25d0f.gif

5)

hello_html_mee82f1.gifhello_html_7d9f89a2.gif





6)

hello_html_m16606c84.gifhello_html_m19c9f02e.gif

7)

hello_html_md554e97.gifhello_html_4d742c60.gif

hello_html_2b63e896.gif

hello_html_3fe81044.gif

hello_html_m5c87bd31.gif


Задание 2. Построить сечение призмы ABCDA1B1C1D1 плоскостью, проходящей через точки P, Q, R

Решение: hello_html_m781b5c80.jpg

  1. Построим след секущей плоскости на плоскость нижнего основания призмы. Рассмотрим грань АА1В1В. В этой грани лежат точки сечения P и Q. Проведем прямую PQ.

  2. Продолжим прямую PQ, которая принадлежит сечению, до пересечения с прямой АВ. Получим точку S1, принадлежащую следу.

  3. Аналогично получаем точку S2 пересечением прямых QR и BC.

  4. Прямая S1S2 - след секущей плоскости на плоскость нижнего основания призмы.

  5. Прямая S1S2 пересекает сторону AD в точке U, сторону CD в точке Т. Соединим точки P и U, так как они лежат в одной плоскости грани АА1D1D. Аналогично получаем TU и RT.

  6. PQRTU – искомое сечение.

Задание 3 Построить сечение параллелепипеда ABCDA1B1C1D1 плоскостью, проходящей через точки M, N, P .

Решение:

hello_html_m540cf348.jpg

  1. Точки N и P лежат в плоскости сечения и в плоскости нижнего основания параллелепипеда. Построим прямую, проодящую через эти точки. Эта прямая является следом секущей плоскости на плоскость основания параллелепипеда.

  2. Продолжим прямую, на которой лежит сторона AB параллелепипеда. Прямые AB и NP пересекутся в некоторой точке S. Эта точка принадлежит плоскости сечения.

  3. Так как точка M также принадлежит плоскости сечения и пересекает прямую АА1 в некоторой точке Х.

  4. Точки X и N лежат в одной плоскости грани АА1D1D, соединим их и получим прямую XN.

  5. Так как плоскости граней параллелепипеда параллельны, то через точку M можно провести прямую в грани A1B1C1D1, параллельную прямой NP. Эта прямая пересечет сторону В1С1 в точке Y.

  6. Аналогично проводим прямую YZ, параллельно прямой XN. Соединяем Z с P и получаем искомое сечение – MYZPNX.

Задание 4. Построить сечение пирамиды плоскостью, проходящей через точки M, N, P.


hello_html_4f3a9bf7.png

Решение: Точки M и N лежат в одной плоскости ABS, следовательно, через них можем провести прямую. След этой прямой — отрезок MN. Он видимый, значит, соединяем M и N сплошной линией.

Точки M и P лежат в одной плоскости ACS, поэтому через них проведем прямую. След — отрезок MP. Мы его не видим, поэтому отрезок MP проводим штрихом. Аналогично строим след PN.

Треугольник MNP — искомое сечение.

hello_html_53c3d8f5.png

Задание 5. Построить сечение пирамиды плоскостью, проходящей через точки M, N, P.

hello_html_m3e755642.jpg

Решение: Точки M и N лежат в одной плоскость ABS, поэтому через них можно провести прямую. Получаем след MN. Аналогично — NP. Оба следа видимые, поэтому соединяем их сплошной линией.

hello_html_m5acaa683.jpg

Точки M и P лежат в разных плоскостях. Поэтому соединить их прямой не можем.

Продолжим прямую NP.

Она лежит в плоскости грани BCS. NP пересекается только с прямыми, лежащими в этой же плоскости. Таких прямых у нас три: BS, CS и BC. С прямыми BS и CS уже есть точки пересечения — это как раз N и P. Значит, ищем пересечение NP с прямой BC.

hello_html_m33703e51.jpg

Точку пересечения (назовем ее H), получаем, продолжая прямые NP и BC до пересечения.

Эта точка H принадлежит как плоскости (BCS), поскольку лежит на прямой NP, так и плоскости (ABC), поскольку лежит на прямой BC.

Таким образом мы получили еще одну точку секущей плоскости, лежащей в плоскости (ABC).

hello_html_7bf80377.jpg

Через H и точку M, лежащую в этой же плоскости, можем провести прямую. Получим след MT. T — точка пересечения прямых MH и AC.

Так как T принадлежит прямой AC, то через нее и точку P можем провести прямую, так как они обе лежат в одной плоскости (ACS).

hello_html_68f594ea.jpg 

Четырёхугольник MNPT — искомое сечение пирамиды плоскостью, проходящей через данные точки M,N,P. 


Задание 6. Построить сечение пирамиды плоскостью, проходящей через точки M, N, P.

hello_html_5e36f99.png

 Решение:

Через точки M и N, лежащие в одной плоскости (BCS), проводим прямую. Получаем след MN (видимый).

Через точки N и P, лежащие в одной плоскости (ACS), проводим прямую. Получаем след PN (невидимый).

hello_html_m254848b8.png

Через точки M и P прямую провести не можем.

1) Прямая MN лежит в плоскости (BCS), где есть еще три прямые: BC, SC и SB. С прямыми SB и SC уже есть точки пересечения: M и N. Поэтому ищем точку пересечения MN с BC. Продолжив эти прямые, получаем точку L. 

hello_html_m17bb0d6e.png

Точка L принадлежит прямой BC, а значит, она лежит в плоскости (ABC). Поэтому через L и P, которая также лежит в плоскости (ABC) можем провести прямую. Ее след — PF.

F лежит на прямой AB, а значит, и в плоскости (ABS). Поэтому через F и точку M, которая также лежит в плоскости (ABS), проводим прямую. Ее след — FM. Четырехугольник MNPF — искомое сечение.

hello_html_3f3f6e2c.png

2) Другой путь — продолжить прямую PN. Она лежит в плоскости (ACS) и пересекается с прямыми AC и CS, лежащими в этой плоскости, в точках P и N.

Значит, ищем точку пересечения PN с третьей прямой этой плоскости — с AS. Продолжаем AS и PN, на пересечении получаем точку E. Поскольку точка E лежит на прямой AS, принадлежащей плоскости (ABS), то через E и точку M, которая также лежит в (ABS), можем провести прямую. Ее след — FM. Точки P и F лежат водной плоскости (ABC), проводим через них прямую и получаем след PF (невидимый).



























Задания для самостоятельного решения

Практическая работа №12

Тема: «Построение сечений многогранников плоскостью»

ABCDA1B1C1D1 плоскостью MNK, проходящей через 3 cсамостоятельно выбранные на ребрах точки M, N, K.

Построить сечения пирамиды SABCD плоскостью MNK, проходящей через 3 самостоятельно выбранные на ребрах точки M, N, K.

*
















1-10
















Практическая работа№13

Тема: «Нахождение элементов многогранников и круглых тел»

Цель работа: формирование у учащихся умений и навыков решения задач на нахождение элементов многогранников и круглых тел, развитие у учащихся пространственного воображения, графической культуры и математической речи.

В результате выполнения практической работы студент должен:

знать:

  • понятие многогранника;

  • какая фигура называется тетраэдром, параллелепипедом, призмой, пирамидой, цилиндром, конусом, шаром;

  • основные элементы многогранников и круглых тел;

уметь:

  • находить элементы многогранников и круглых тел;

  • строить сечение пирамиды плоскостью проходящей через три точки;


Краткие теоретические сведения

1 Понятие многогранника.

Поверхность, составленную из многоугольников и ограничивающую некоторое геометрическое тело называют многогранником. Тело, ограниченное многогранником, часто также называют многогранником.

Многоугольники, из которых составлен многогранник, называются его гранями. Стороны граней называют ребрами, а концы ребер – вершинами многогранника. Отрезок, соединяющий две вершины, не принадлежащие одной грани, называется диагональю многогранника. Плоскость, по обе стороны от которой имеются точки многогранника, называется секущей плоскостью, а общая часть многогранника и секущей плоскости – сечением многогранника.hello_html_m4dc99857.pnghello_html_41e9967c.png

Многогранники бывают выпуклые – если они расположены по одну сторону от плоскости каждой его грани, и невыпуклые. В выпуклом многограннике сумма всех плоских углов при каждой его вершине меньше .

Выпуклый многогранник называется правильным, если все его грани – равные правильные многоугольник и в каждой его вершине сходится одно и тоже число ребер. Не существует правильного многогранника, гранями которого являются правильные n-угольники при .

2 Тетраэдр

hello_html_41e9967c.png

Поверхность, составленная из четырех равносторонних треугольников ABC, DAB, DBC, DCA называется тетраэдром DABC.

Треугольники, из которых состоит тетраэдр называются гранями, их стороны – ребра, а вершины – вершины тетраэдра. Тетраэдр имеет четыре грани, шесть ребер и четыре вершины. Два ребра тетраэдра, не имеющие общих вершин, называются противоположными (AD и BC, BD и AC, CD и AB. Одна из граней тетраэдра называется его основанием, а три другие – боковыми гранями. Сумма плоских углов при каждой вершине равна


3 Параллелепипед

Поверхность, составленная из двух равных параллелограммов и называется параллелепипедом . Параллелограммы, из которых составлен параллелепипед, называют его гранями, их стороны – ребрами, а вершины параллелограммов – вершинами параллелепипеда. Параллелепипед имеет шесть граней, двенадцать ребер и восемь вершин. Две грани параллелепипеда, имеющие общее ребро, называются смежными, а не имеющие общих ребер – противоположными. hello_html_m4dc99857.png

Отрезок, соединяющий противоположные вершины, называется диагональю параллелепипеда. Каждый параллелепипед имеет четыре диагонали. Часто выделяют какие-нибудь две противоположные грани и называют их основаниями, а остальные грани – боковыми гранями параллелепипеда. Ребра параллелепипеда, не принадлежащие основаниям, называются боковыми ребрами.hello_html_m2b85a131.jpg

Свойства параллелепипеда:

  1. Противоположные грани параллелепипеда параллельны и равны.

  2. Диагонали параллелепипеда пересекаются в одной точке и делятся этой точкой пополам.



  1. Призма


Многогранник, составленный из двух равных многоугольников и , расположенных в параллельных плоскостях, и n параллелограммов, называется призмой. Многогранники и называются основаниями, а параллелограммы – боковыми гранями призмы. Отрезки , , …, называются боковыми ребрами призмы. Эти ребра равны и параллельны. Призму с основаниями и обозначают и называют n-угольной призмой. Перпендикуляр, проведенный из какой-нибудь точки одного основания к плоскости другого основания, называется высотой призмы. Если боковые ребра призмы перпендикулярны к основаниям, то призма называется прямой, в противном случае – наклонной. Высота прямой призмы равна ее боковому ребру. Прямая призма называется правильной, если ее основания – правильные многоугольники. hello_html_540f99ef.gif


5 Пирамида

Многогранник, составленный из n-угольника и n треугольников, называется пирамидой. Многоугольник называется основанием, а треугольники – боковыми гранями пирамиды. Точка Р называется вершиной пирамиды, а отрезки , – ее боковыми ребрами. Перпендикуляр, проведенный из вершины пирамиды к плоскости основания, называется высотой пирамиды.hello_html_m30e579ad.png


Пирамида называется правильной, если ее основание – правильный многоугольник, а отрезок, соединяющий вершину пирамиды с центром основания, является ее высотой. Все боковые ребра правильной пирамиды равны, а боковые грани являются равными равнобедренными треугольниками. Высота боковой грани правильной пирамиды, проведенная из вершины, называется апофемой.hello_html_32bb6818.png

Теорема. Площадь боковой поверхности правильной пирамиды равна половине произведения периметра основания на апофему.


Многогранник, гранями которого являются n-угольники и (нижнее и верхнее основания), расположенные в параллельных плоскостях, и n четырехугольников (боковые грани), называется усеченной пирамидой. Отрезки называются боковыми ребрами усеченной пирамиды. Перпендикуляр, проведенный из какой-нибудь точки одного основания к плоскости другого основания, называется высотой усеченной пирамиды. Боковые грани усеченной пирамиды – трапеции. Усеченная пирамида называется правильной, если она получена сечением правильной пирамиды плоскостью, параллельной основанию. Основания правильной усеченной пирамиды – правильные многоугольники, а боковые грани – равнобедренные трапеции. Высоты этих трапеций называются апофемами. Площадью боковой поверхности усеченной пирамиды называется сумма площадей ее боковых граней.hello_html_18cc931e.png


6 Цилиндр

Поверхность, образованная прямыми, называется цилиндрической поверхностью, а сами прямые – образующими цилиндрической поверхности. Прямая, проходящая через точку О перпендикулярно плоскости, называется осью цилиндрической поверхности. Все образующие и ось перпендикулярны к плоскости, то они параллельны друг другу. Тело, ограниченное цилиндрической поверхностью и двумя кругами с границами, называется цилиндром. Круги называются основаниями цилиндра, отрезки образующих, заключенные между основаниями, - образующими цилиндра, а образованная ими часть цилиндрической поверхности – боковой поверхностью цилиндра. Ось цилиндрической поверхности называется осью цилиндра. Длина образующей называется высотой цилиндра, а радиус основания – радиусом цилиндра. Цилиндр может быть получен вращением прямоугольника вокруг одной из его сторон. hello_html_m29c71cb6.png

Если секущая плоскость проходит через ось цилиндра, то сечение представляет собой прямоугольник. Такое сечение называется осевым. Если секущая плоскость перпендикулярна к оси цилиндра, то сечение является кругом.

Разверткой боковой поверхности цилиндра является прямоугольник.



  1. Конус

Поверхность, образованная прямыми, называется конической поверхностью, а сами прямые – образующими конической поверхности. Точка Р называется вершиной, а прямая, проведенная из вершины к плоскости основания – осью конической поверхности. Тело, ограниченное конической поверхностью и кругом с границей, называется конусом. Круг называется основанием конуса, вершина конической поверхности – вершиной конуса, отрезки образующих, заключенные между вершиной и основанием, - образующими конуса, а образованная ими часть конической поверхности – боковой поверхностью конуса. Ось конической поверхности называется осью конуса, а ее отрезок, заключенный между вершиной и основанием, - высотой конуса. Все образующие конуса равны друг другу. Конус может быть получен вращением прямоугольного треугольника вокруг одного из его катетов.hello_html_5013a398.png

Если секущая плоскость проходит через ось конуса, то сечение представляет собой равнобедренный треугольник, основание которого – диаметр основания конуса. Такое сечение – осевое. Если секущая плоскость перпендикулярна к оси конуса, то сечение конуса представляет собой круг с центром, расположенным на оси конуса.

Если возьмем произвольный конус и проведем секущую плоскость, перпендикулярную к его оси, то эта плоскость пересечется с конусом по кругу и разобьет его на две части. Одна из частей (верхняя) представляет собой конус, а другая называется усеченным конусом. Основание исходного конуса и круг, полученный в сечении этого конуса плоскостью, называются основаниями усеченного конуса, а отрезок, соединяющий их центры, - высотой усеченного конуса. Часть конической поверхности, ограничивающая усеченный конус, называется его боковой поверхностью, а отрезки образующих конической поверхности, заключенные между основаниями, называются образующими усеченного конуса. hello_html_63b3edfc.png


  1. Сфера и шар

Сферой называется поверхность, состоящая из всех точек пространства, расположенных на данном расстоянии от данной точки. Данная точка называется центром сферы, а расстояние – радиусом сферы. Отрезок, соединяющих две точки сферы и проходящий через ее центр, называется диаметром сферы. Тело, ограниченное сферой, называется шаром. Центр, радиус и диаметр сферы называются также центром, радиусом и диаметром шара. hello_html_403111ac.jpg

В прямоугольной системе координат уравнение сферы радиуса R с центром : .









Образец решения задач


Задача 1. Образующая конуса, равная 12 см, наклонена к плоскости основания под углом . Найдите радиус основания конуса, если: а) , б) , в) .

Решение:

а) hello_html_4d9473e9.png


б)



в)



Ответ.


Задача 2. Осевое сечение цилиндра – квадрат, диагональ которого равна 20 см. Найдите высоту цилиндра.

Решение:

Осевое сечение цилиндра – квадрат , hello_html_m24e0ee69.jpg


По теореме Пифагора, т.е.





т. е.

Ответ.

Задача 3. Сумма всех ребер параллелепипеда равна 120 см. Найдите каждое ребро параллелепипеда, если

Решение:

Т.к. у параллелепипеда все боковые ребра равны, то пусть . Тогда hello_html_m4dc99857.png

Из условия следует, что:






Получаем

Ответ.


Задача 4. Основанием пирамиды является ромб, сторона которого равна 5 см, а одна из диагоналей равна 8 см. Найдите боковые ребра пирамиды, если высота ее проходит через точку пересечения диагоналей основания и равна 7 см.

Решение:

Точка пересечения диагоналей является центром ромба . Следовательно пирамида является правильной. По свойству диагоналей ромба: hello_html_m4ea9b74.png

Из треугольника ASO по теореме Пифагора: (см).

Из треугольника SDO по теореме Пифагора:

SA=SC= см и SB=SD= см как наклонные, имеющие одинаковые проекции.

Ответ. SA=SC= см, SB=SD=




























Задания для самостоятельного решения

Практическая работа №13

Тема: «Нахождение элементов многогранников и круглых тел»


2. Высота призмы 14 см. Найти площадь ее диагонального сечения.

Через вершину основания правильной треугольной пирамиды и апофему противолежащей боковой грани проведено сечение. Найти площадь сечения, если длина стороны основания 12 м, а высота пирамиды 2 м.

Прямоугольник диагональ которого равна 25 см, а одна сторона 20 см вращается вокруг меньшей стороны. Вычислить длину высоты полученного цилиндра и площадь его основания.

Длины радиусов оснований и образующий усеченного конуса равен соответственно 7 см.,15 см. и 17см.Найти высоту конуса.

Угол между двумя радиусами шара 90º.Расстояние между их концами 15 см. Найти расстояние по поверхности шара между концами радиусов.

2

Диагональ боковой грани правильной треугольной призмы равна 6 дм и наклонная к плоскости основания под углом 30º.Найти площадь основание призмы.

Через вершину правильной шестиугольной пирамиды и диаметр окружности описанной около ее основания проведено сечение. Вычислить площадь сечения, если сторона оснований пирамиды равное 4 см, а ее высота 5 см.

Осевое сечение цилиндра – квадрат, диагональ которого 12 см. Вычислить длину образующей и площадь основания цилиндра.

Диаметр шара 52 см. Вычислить площадь сечения шара плоскостью, удаленной от его центра на 10 см..

Точка С сферы удалена от концов его диаметра – АВ на 10 см и 24 см. Вычислить длину линии пересечения сферы и плоскости содержащей точки А,В и С.

3

Набольшая диагональ правильной шестиугольной призмы равна 12 см, она наклонная к плоскости основания под углом 60º. Вычислить длину стороны основания призмы.

Через вершину и диагональ основания правильной четырех угольной пирамиды проведено сечение. Вычислите его площадь, если сторона основания равное 8 см, а боковое ребро пирамиды 5 см.

Диагональ прямоугольника 18 см, она составляет с его стороной угол 30º. Прямоугольник вращается большей стороны. Вычислить высоту и площадь основания полученного цилиндра.

Длины радиусов оснований усеченного конуса 10 см и 8 см. Угол между образующей и плоскостью основания 45º. Вычислить площадь осевого сечения усеченного конуса.

На поверхности шара даны три точки, расстояние между которыми равны 8 см. Вычислить площадь сечения шара плоскостью содержащей эти точки.

4

Основание прямой призмы – ромб со стороной 8 см и острым углом 60º. Высота призмы 12 см. Вычислить длины диагоналей призмы.

Через вершину и середины двух соседних сторон основания правильной четырехугольной пирамиды поведено сечение. Найти его периметр, если сторона основания пирамиды 8м,а боковое ребро5м.

Высота цилиндра на 2 см больше радиуса его основания. Площадь осевого сечения цилиндра 96 см2. Найти радиус основания и высоту цилиндра.

Длины радиусов оснований и высота конуса равны соответственно 4 дм, 20 дм и 30 дм. Найти длину образующей этого усеченного конуса.

Радиус сферы 10 см. На расстоянии 5см от ее центра проведена плоскость. Вычислить длину линии их пересечения.

5

Основание прямой пирамиды – прямоугольный треугольник, катеты которого равны 7 см и 24 см. Угол между диагональю большей боковой грани и плоскости основания призмы 45º. Найти высоту призмы.

Сторона основания правильной четырехугольной пирамиды равно 6 м угол между боковым ребром и плоскостью основания 30º. Найти площадь сечения проведенного через два боковых ребра, не лежащих в одной грани.

Образующая цилиндра в 3 раза больше диаметра его основания. Площадь осевого сечения цилиндра 300 см2. Вычислить длину образующей и площадь основания цилиндра.

Длины образующей и диаметра основания конуса равны соответственно 26 см и 20 см. Через середину образующей конуса проведена плоскость параллельная плоскости основания. Найти высоту полученного усеченного конуса.

Диаметр шара 52 см. Вычислить площадь сечения шара плоскостью, удаленной от его центра на 10 см.

6

Основание прямой призмы – равнобедренный треугольник АВС, где АВ=ВС=3. Высота призмы 6 см. Диагональ боковой грани с ребром ВС равна 15 см. Найти площадь основания призмы.

Все ребра правильной четырехугольной пирамиды равны 10 см. Найти периметр сечения, содержащего точки В, D и середину бокового ребра КС пирамиды KABCD.

Осевое сечение цилиндра – квадрат, площадь которого 36 дм2. Вычислить длину образующей и площадь основания цилиндра.

Длины диаметров оснований и образующей усеченного конуса равны соответственно 10 см, 22 см, 20 см. Вычислить площадь осевого сечения усеченного конуса.

Радиус шара 10 см. На расстоянии 8 см от его центра проведена плоскость. Вычислить площадь фигуры, являющихся их пересечением.

7

Площадь основания правильной четырехугольной призмы 625 см2. Высота призмы 14 см. Найти площадь ее диагонального сечения.

Через вершину и диагональ основания правильной четырех угольной пирамиды проведено сечение. Вычислите его площадь, если сторона основания равное 8 см, а боковое ребро пирамиды 5 см.

Длины радиусов оснований и образующий усеченного конуса равен соответственно 7 см.,15 см. и 17 см.Найти высоту конуса.

Угол между двумя радиусами шара 90º.Расстояние между их концами 15 см. Найти расстояние по поверхности шара между концами радиусов.

Осевое сечение цилиндра – квадрат, диагональ которого 12 см. Вычислить длину образующей и площадь основания цилиндра.

8

Диагональ боковой грани правильной треугольной призмы равна 6 дм и наклонная к плоскости основания под углом 30º.Найти площадь основание призмы.

Через вершину и середины двух соседних сторон основания правильной четырехугольной пирамиды поведено сечение. Найти его периметр, если сторона основания пирамиды 8 м, а боковое ребро 5 м.

Длина образующей усеченного конуса 12 см. Длины окружностей его оснований 14П см и 30П см. Найти угол между образующей и плоскостью основания усеченного конуса.

Точка С сферы удалена от концов его диаметра – АВ на 10 см и 24 см. Вычислить длину линии пересечения сферы и плоскости содержащей точки А,В и С.

Диагональ прямоугольника 18 см, она составляет с его стороной угол 30º. Прямоугольник вращается большей стороны. Вычислить высоту и площадь основания полученного цилиндра.

9

Набольшая диагональ правильной шестиугольной призмы равна 12 см, она наклонная к плоскости основания под углом 60º. Вычислить длину стороны основания призмы.

Сторона основания правильной четырехугольной пирамиды равно 6 м угол между боковым ребром и плоскостью основания 30º. Найти площадь сечения проведенного через два боковых ребра, не лежащих в одной грани.

Длины радиусов оснований усеченного конуса 10 см и 8 см. Угол между образующей и плоскостью основания 45º. Вычислить площадь осевого сечения усеченного конуса.

На поверхности шара даны три точки, расстояние между которыми равны 8 см. Вычислить площадь сечения шара плоскостью содержащей эти точки.

Высота цилиндра на 2 см больше радиуса его основания. Площадь осевого сечения цилиндра 96 см2. Найти радиус основания и высоту цилиндра.

10

Основание прямой призмы – ромб со стороной 8 см и острым углом 60º. Высота призмы 12 см. Вычислить длины диагоналей призмы.

Все ребра правильной четырехугольной пирамиды равны 10 см. Найти периметр сечения, содержащего точки В, D и середину бокового ребра КС пирамиды KABCD.

Длины радиусов оснований и высота конуса равны соответственно 4 дм, 20 дм и 30 дм. Найти длину образующей этого усеченного конуса.

Радиус сферы 10 см. На расстоянии 5см от ее центра проведена плоскость. Вычислить длину линии их пересечения.

Образующая цилиндра в 3 раза больше диаметра его основания. Площадь осевого сечения цилиндра 300 см2. Вычислить длину образующей и площадь основания цилиндра.


















Практическая работа №14

Тема: “Вычисление пределов”

Цель работы: научиться вычислять пределы.

В результате выполнения практической работы студент должен:

знать:

  • определения предела последовательности и предела функции;

  • свойства предела:

  • методы вычисления пределов;

уметь:

  • вычислять пределы;

  • раскрывать неопределённости вида .


Краткие теоретические сведения


Число называется пределом последовательности , ,…,,…, если для

всякого сколь угодно малого положительного числа найдется такое положительное число N, что при n>N. В этом случае пишут .

Число A называется пределом функции f(x) при xa, если для любого сколь угодно малого ε>0 найдется такое δ>0, что при . Это записывают так:

.

Аналогично если при .

Условно записывают если при , где M-произвольное положительное число. В этом случае функция f(x) называется бесконечно большой при xa.

Если , то функция a(x) называется бесконечно малой при xa.

Если x и x→a, то употребляют запись xa -0; если x>a и xa - запись xa +0.

Практическое вычисление пределов основывается на следующих теоремах.

Если существуют и , то




  1. (при ).







Образец решения задач


Вычислить пределы.







  1. (разделим числитель и знаменатель на )











Задания для самостоятельной работы

Практическая работа №14.

Тема: «Вычисление пределов»

Практическая работа №15

Тема: “Исследование функций с помощью производной”


Цель работы: научиться исследовать функции по общей схеме и строить графики.


В результате выполнения работы студент должен:

знать:

  • что называется областью определения функции;

  • какая функция называется возрастающей (убывающей);

  • необходимое условие экстремума функции;

  • определение точки перегиба;

  • определение интервалов выпуклости графика функции;

  • определение асимптот графика функции;

уметь:

  • находить область определения функции;

  • находить точки пересечения графика функции с осями координат;

  • применять первую производную для нахождения точек экстремума и промежутков монотонности;

  • применять вторую производную для нахождения точек перегиба и направления выпуклости графика функции;

  • находить асимптоты графика функции;

  • строить график функции.


Краткие теоретические сведения


1 Областью определения функции называется множество значений переменной , при которых данная функция имеет смысл.

2 Возрастание и убывание функции. Экстремум функции. Функции f(x) называется возрастающей в точке , если при любом достаточно малом h>0 выполняется условие (рис. 1).

hello_html_62171edf.png

Функция f(x) называется убывающей в точке , если при любом достаточно малом h>0 выполняется условие (рис.2).

Функция f(x) называется возрастающей в интервале ]a, b[, если для любых двух точек и из указанного интервала, удовлетворяющих неравенству <, выполняется неравенство .

Функция f(x) называется убывающей в интервале ]a, b[, если для любых точек и из указанного интеграла, удовлетворяющих неравенству , выполняется неравенство .

hello_html_39ba7f24.png

Признаки возрастания и убывания функции.

  1. Если , то функция f(x) возрастает в точке .

  2. Если , то функция f(x) убывает в точке .

Значение называется максимумом функции f(x), если при любом достаточно малом h>0 выполняются условия и . Точка называется в этом случае точкой максимума функции f(x) (рис. 3).

hello_html_2282937d.png

Значение называется минимумом функции f(x), если при любом достаточно малом h>0 выполняются условия и . Точка называется в этом случае точкой минимума функции f(x) (рис. 4).

hello_html_m70aee991.png

Максимум или минимум функции называется экстремумом функции. Точка максимума или минимума функции называется точкой ее экстремума.

Необходимое условие экстремума. Если функция f(x) в точке имеет экстремум, то производная f'() обращается в нуль или не существует.

Точка , в которой , называется стационарной точкой. Точки, в которых или не существует, называются критическими точками. Не всякая критическая точка является точкой экстремума.

Достаточные условия экстремума.

Правило 1. Если -критическая точка функции f(x) и при произвольном достаточно малом h>0 выполняются неравенства , , то функция f(x) в точке имеет максимум; если же , , то функция f(x) в точке имеет минимум.

Если знаки и одинаковы, то функция f(x) в точке экстремума не имеет.

Правило 2. Если , , то функция f(x) в точке имеет экстремум, а именно максимум, если , и минимум, если .

Правило 3. Пусть , ,…, , . В этом случае функция f(x) имеет в точке экстремум, если n-четное число, а именно максимум при и минимум при , если же n-нечетное число, то функция f(x) в точке экстремума не имеет.

Алгоритм нахождения промежутков монотонности и точек экстремума:

1 Находим ;

2 Решая уравнение , находим критические точки;

3 Наносим критические точки на числовую ось и определяем знак на каждом числовом

промежутке;

4 Там, где функция возрастает, а где функция убывает;

5 Если при переходе через критическую точку ( там, где ) производная меняет свой знак,

то - абсцисса точки экстремума.

3 Выпуклость графика функции. Точки перегиба.

Определение: График непрерывно дифференцируемой функции ,

называется выпуклым вверх на интервале , если производная убывает на А если возрастает на , то график этой функции называется выпуклым вниз.


У У









X X

Легко видеть, что если график функции выпуклый вверх, то все его точки лежат ниже любой его касательной (рис.5,а), так как угловой коэффициент касательной уменьшается с возрастанием .

А если график выпуклый вниз (рис.5,б), то все точки лежат выше любой его касательной (кроме, конечно, самой точки касания).


Определение: Интервалы, на которых график функции выпуклый вверх или вниз, называются интервалами выпуклости графика функции.


Достаточное условие выпуклости графика функции.

Теорема: Пусть функция , имеет первую и вторую производные. Тогда, если для всех , то на интервале график функции выпуклый вверх, если же для всех , то график функции выпуклый вниз на .

Условие знакопостоянства второй производной, является достаточным условием выпуклости (вверх или вниз) графика функции, не является вместе с тем необходимым условием.


Определение: Точка графика дифференцируемой функции, абсцисса которой является одновременно концом интервала выпуклости вверх и концом интервала выпуклости вниз, называется точкой перегиба графика функции.


Очевидно, что в точке перегиба касательная к графику кривой должна, с одной стороны, находится выше графика кривой, а с другой,- ниже его, т.е. пересекать кривую в этой точке (Рис.6)


У








0 Х


Теорема: (необходимое условие). Пусть функция на интервале имеет непрерывную производную второго порядка. Тогда, если точка с абсциссой является точкой перегиба графика функции, то .

Теорема: (достаточное условие). Пусть функция на интервале имеет производную второго порядка. Тогда, если меняет знак при переходе аргумента через , то является абсциссой точки перегиба данной функции.


Алгоритм нахождения промежутков выпуклости и точек перегиба:

1 Находим ;

2 Решая уравнение , находим критические точки;

3 Наносим критические точки на числовую ось и определяем знак на каждом числовом

промежутке;

4 Если , то на этом интервале график функции выпуклый вниз, если же , то

выпуклый вверх.

5 Если при переходе через критическую точку ( там, где ) производная меняет свой знак, то

- абсцисса точки перегиба


4 Асимптоты графика функции.


Асимптоты это прямые, к которым неограниченно приближается график функции.

Различают 3 вида асимптот: вертикальные, горизонтальные и наклонные.

Вертикальная асимптота графика функции - это прямая . Обычно эти асимптоты сопровождают точки разрыва 2-го рода. Если функция непрерывна, то вертикальных асимптот нет.

Наклонная асимптота графика функции - это прямая , если

и .

Вертикальная асимптота графика функции - это прямая , если .


Образец решения задач


Провести полное исследование и построить график функции.

Пример 1.


Решение:


1) Найдём область определения функции:


2) Найдём точки пересечения графика функции с осями координат:

А) с осью ОХ: у=0

. Сгруппируем:

Т.е. (1;0) – точка пересечения с осью ОХ.


Б) с осью ОУ: х=0

Т.е. (0;-1) - точка пересечения с осью ОУ.


3) Асимптот нет, так как нет точек разрыва функции.


4) Исследуем функцию на монотонность и найдём точки экстремума:


, /:3

критическая точка функции по первой производной.

Отметим эту точку на координатной прямой и найдём знак в первой производной в каждом из интервалов, на которые разбивается координатная прямая данной точкой.


+

----------------------------------------------------------------------


Т.е. функция возрастает при . Экстремума нет.

5) Исследуем функцию на выпуклость и найдём точки перегиба:


критическая точка функции по второй производной.

Отметим эту точку на координатной прямой и найдём знак второй производной в каждом из интервалов, на которые разбивается координатная прямая данной точкой.


- 1 +

----------------------------------------------------------------------------------

т.п.

Т.е. функция выпукла вверх при ,

функция выпукла вниз при

, т.е. (1;0)-точка перегиба.


6) Построим график функции:

hello_html_7d07bd.png






























Задачи для самостоятельного решения

Практическая работа №15

Тема: “Исследование функций с помощью производной”


Провести полное исследование и построить график функции.

б)

3

а) ;

б)

4

а) ;

б)

5

а)

б)

6

а) ;

б)

7

а) ;

б)


8

а) ;

б)

9

а);

б)

10

а) ;

б)










































Практическая работа №16

Тема: «Вычисление площадей плоских фигур»


Цель работы: научиться вычислять площади плоских фигур с помощью определённого интеграла.


В результате выполнения практической работы студент должен:

знать:

  • формулу Ньютона-Лейбница;

  • геометрический смысл определённого интеграла;

  • формулы для вычисления площадей плоских фигур;

уметь:

  • вычислять площади плоских фигур.


Краткие теоретические сведения


1 Определённый интеграл


Пусть функция f(x) определена на отрезке [a, b]. Разделим отрезок [a, b] на n произвольных частей точками , выберем на каждом элементарном отрезке произвольную точку и найдем длину каждого такого отрезка: .

Интегральной суммой для функции f(x) на отрезке [a, b] называется сумма вида , причем эта сумма имеет конечный предел I, если для каждого найдется такое число , что при неравенство выполняется при любом выборе чисел .

Определенным интегралом от функции f(x) на отрезке [a, b] (или в пределах от a

до b) называется предел интегральной суммы при условии, что длина наибольшего из элементарных отрезков () стремится к нулю:

Если функция f(x) непрерывна на [a, b], то предел интегральной суммы существует

и не зависит от способа разбиения отрезка [a, b] на элементарные отрезки и от выбора точек (теорема существования определенного интеграла).

Числа a и b соответственно называются нижним и верхним пределами интегрирования.


Если f(x)>0 на [a, b], то определенный интеграл геометрически представляет собой площадь криволинейной трапеции-фигуры, ограниченной линиями y=f(x), x=a, x=b, y=0.

hello_html_m1455aa58.png

Основные свойства определенного интеграла


10. .

20. .

30.

40.

50. , c -постоянная.


Формула Ньютона-Лейбница



где F(x)-первообразная для f(x), т.е. F’(x)=f(x).



2 Вычисление площади плоской фигуры


Площадь криволинейной трапеции , ограниченной кривой y=f(x) [f(x)≥0], прямыми x=a и x=b и отрезком [a,b] оси Ox, вычисляется по формуле:

hello_html_m6d94f654.png


Площадь фигуры, ограниченной кривыми , и прямыми x=a, x=b

находится по формуле: .

hello_html_m5e3eb1bb.png

Образец решения задач


Задание 1. Вычислить площадь фигуры, ограниченной линиями:

Решение:

Построим прямые:


hello_html_m427449fb.png



Ответ:


Задание 2. Вычислить площадь фигуры, ограниченной линиями:

Решение:

1) Построим параболу . Найдём вершину параболы:


Подставим в уравнение параболы и найдём .

Т.е. вершина параболы находится в точке (2;4), ветви параболы направлены вниз. В сдвинутых осях строим параболу .

hello_html_m4247447f.png


2) Построим прямую .

3) Найдём пределы интегрирования:



4)

Ответ:

Задание 3. Вычислить площадь фигуры, ограниченной линиями:

Решение:


  1. Построим параболу и прямую

hello_html_50b078fb.png


  1. Найдём пределы интегрирования:



Ответ:


























Задания для самостоятельного решения

Практическая работа №16

Тема: «Вычисление площадей плоских фигур»

Вычислить площадь фигуры, ограниченной линиями


1

2

3

1


у = 2x + 4, у = 4 – х, у = 0


, у = х + 3


ху = 4, у = 5 – х

2



у = 3х + 3, у = -2х + 4, у = 0


, у = х + 3

, у = х + 7

3



у = х + 5, у = 7 – х, у = 0


, у = х - 1


ху = 3, у = 4 - х

4



у = 3 – х, у = 2х + 6, у = 0


, у = х + 5

, у = - х + 6

5



у = - х + 4, у = 8 + 3х, у = 0


, у = х + 5


ху = - 4, у = х + 5

6



у = 4х + 2, у = 12 – х, у = 0


, у = х + 6

, у = 5 - х

7



у = х + 1, у = 4 – 2х, у = 0


, у = х - 1

, у = 3 – х

8



у = 5х + 5, у = 11 – х, у = 0


, у = х

, у = х + 4

9



у = х + 4, у = -х + 6, у = 0


, у = х + 3


ху = 6, у = 7 - х

10



у = 3х + 6, у = 10 – х, у = 0


, у = х - 3


ху = - 5, у = х + 6

Практическая работа №17

Тема: «Вычисление вероятности события»


Цель работы: научиться вычислять вероятности событий.


В результате выполнения практической работы студент должен:

знать:

  • определение вероятности события;

  • формулы комбинаторики;

уметь:

  1. вычислять вероятности событий.


Краткие теоретические сведения



Вероя́тность — степень (относительная мера, количественная оценка) возможности наступления некоторого события.

Вероятностью события A называют отношение числа m благоприятствующих этому событию исходов к общему числу n всех равновозможных несовместных элементарных исходов, образующих полную группу
hello_html_m649d0d41.gif

Свойство1. Вероятность достоверного события равна единице
Свойство 2. Вероятность невозможного события равна нулю.
Свойство 3. Вероятность случайного события есть положительное число, заключенное между нулем и единицей.

 hello_html_m793ceb38.gif.

Свойство 4. Сумма вероятностей противоположных событий равна единице.

Образец решения задач

Задача 1. В вазе имеется 9 цветов: 5 роз и 4 гвоздики. Взяли 3 цветка.

Найти вероятность того, что

а) все три цветка розы;

б) взяты розы или гвоздики;

в) из трёх взятых цветов одна роза и две гвоздики.


Решение:

а) А - Все три цветка розы.

n- число всех исходов (взяли 3 цветка из 9),



m-число благоприятных исходов (все три цветка розы: взяли 3 цветка из 5 роз),

или 11,9%

Ответ:


б) В - Взяты розы или гвоздики.

n- число всех исходов (взяли 3 цветка из 9),

m-число благоприятных исходов (взяты розы или гвоздики: взяли 3 цветка из 5 роз или 3 цветка из 4 гвоздик),

или 16,7%

Ответ:

в) Из трёх взятых цветов одна роза и две гвоздики.

n- число всех исходов (взяли 3 цветка из 9),

m-число благоприятных исходов (из трёх взятых цветов одна роза и две гвоздики: взяли 1 цветок из 5 роз и 2 цветка из 4 гвоздик),

или 35,7%

Ответ:


Задача 2. В группе 30 человек, из которых 12 девочек. Для участия в соревнованиях выбрали 5 человек. Найти вероятность того, что

а) выбраны все мальчики;

б) в соревнованиях участвовали три девочки.

Решение:

а) Выбраны все мальчики.

n- число всех исходов (взяли 5 человек из 30),

m-число благоприятных исходов (выбраны все мальчики: взяли 5 мальчиков из 18)






б) В соревнованиях участвовали 3 девочки (а значит и 2 мальчика).

n- число всех исходов (взяли 5 человек из 30),

m-число благоприятных исходов (в соревнованиях участвовали 3 девочки, а значит и 2 мальчика: взяли 3 девочки из 12 и 2 мальчика из 18)


Ответ: а)

б)


Задача 3. Из 500 деталей, среди которых 100 бракованных, наугад берутся 2 детали. Какова вероятность того, что из двух взятых деталей одна бракованная?

Решение:

n- число всех исходов (взяли 2 детали из 500),

m-число благоприятных исходов (взяли 1 деталь из 100 бракованных и 1 деталь из 400 годных)


Ответ:


Задача 4. Некто, набирая номер телефона, забыл последнюю цифру. Какова вероятность того, что набирая её случайным образом, он правильно наберёт номер?

Решение:

n- число всех исходов (всего 10 цифр),

m-число благоприятных исходов (1 единственный вариант),

Ответ:







Задача 5. Телефонный номер состоит из шести цифр, две из которых неизвестны. Найти вероятность того, что

а) были угаданы обе цифры;

б) обе угаданные цифры различны.


Решение:

а) Были угаданы обе цифры (и первая, и вторая).


n- число всех исходов (у одной цифры 10 вариантов и у другой цифры 10 вариантов) , т. е.

m-число благоприятных исходов (1 единственный вариант),


б) Обе угаданные цифры различны.


n- число всех исходов (у одной цифры 10 вариантов, а у другой цифры 9 вариантов) , т. е.


m-число благоприятных исходов (1 единственный вариант),


Ответ: а)

б)


Задача 6. На перекрёстке 4 светофора. Какова вероятность того, что везде горит жёлтый свет?

Решение:

n- число всех исходов (у каждого светофора 3 варианта света: зелёный, жёлтый, красный) , т. е.

m-число благоприятных исходов (у каждого светофора горит жёлтый свет ),

Ответ:

Задача 7. Из цифр от 0 до 9 выбирают две. Какова вероятность того, что

а) сумма цифр равна 5;

б) сумма цифр делится на 10?

Решение:



а) Сумма цифр равна 5.

n- число всех исходов (взяли 2 цифры из 10) ,

m-число благоприятных исходов (сумма цифр равна 5: 1+4; 4+1; 2+3, 3+2), т. е.

б) Сумма цифр делится на 10.

n- число всех исходов (взяли 2 цифры из 10) ,

m-число благоприятных исходов (сумма цифр делится на 10: 1+9; 9+1; 2+8; 8+2; 3+7; 7+3; 4+6; 6+4; 5+5),

Ответ: а)

б)


Задача 8. В первом ящике находятся шары с номерами 1, 2, 3, 4, 5, во втором с номерами 6, 7, 8, 9, 10. Из каждого ящика взяли по одному шару. Какова вероятность того, что

а) сумма очков делится на 11;

б) сумма очков меньше или равна 11?

Решение:

а) Сумма очков делится на 11.

n- число всех исходов (из каждого ящика взяли по одному шару: 1 из первого и 1 из второго) ,

m-число благоприятных исходов (сумма очков делится на 11: 1+10; 2+9; 3+8, 4+7; 5+6), т. е.

б) Сумма очков меньше или равна 11.

n- число всех исходов (из каждого ящика взяли по одному шару: 1 из первого и 1 из второго) ,

m-число благоприятных исходов (сумма очков меньше или равна 11:

1+6; 1+7; 1+8; 1+9; 1+10; 2+6; 2+7; 2+8 2+9; 3+6; 3+7; 3+8; 4+6; 4+7; 5+6),

Ответ: а)

б)






Задача 9. В ящике 10 шаров: 6 белых и 4 чёрных. Из ящика взяли 2 шара. Какова вероятность того, что

а) оба шара белые;

б) оба белые или оба чёрные;

в) один шар белый и один шар чёрный;

г) хотя бы один белый?

Решение:

а) Оба шара белые.

n- число всех исходов (взяли 2 шара из 10) ,

m-число благоприятных исходов (оба шара белые: взяли 2 шара из 6 белых),

б) Оба белые или оба чёрные.

n- число всех исходов (взяли 2 шара из 10) ,

m-число благоприятных исходов (оба белые или оба чёрные: взяли 2 шара из 6 белых или 2 шара из 4 чёрных),

в) Один шар белый и один шар чёрный.

n- число всех исходов (взяли 2 шара из 10) ,

m-число благоприятных исходов (один шар белый и один шар чёрный :взяли 1 шар из 6 белых и 1 шара из 4 чёрных),

г) Хотя бы один белый.

n- число всех исходов (взяли 2 шара из 10) ,

m-число благоприятных исходов (хотя бы один белый: или 2 белых, или 1 белый и 1 чёрный),

Ответ: а)

б)

в)

г)




Задача 10. В лотерее 1000 билетов, из них 100 выигрышных. Купили 3 билета. Какова вероятность, что

а) один выигрышный;

б) хотя бы один выигрышный?

Решение:

а) Один выигрышный.

n- число всех исходов (взяли 3 билета из 1000) ,

m-число благоприятных исходов (один выигрышный, а значит 2 проигрышных: взяли 1 билет из 100 и 2 билета из 900),

б) Хотя бы один выигрышный. Найдём вероятность того, что все 3 билета проигрышные.

n- число всех исходов (взяли 3 билета из 1000) ,

m-число благоприятных исходов (все 3 билета проигрышные: взяли 3 билета из 900),

Искомая вероятность равна:

Ответ: а) ; б)


Задача 11. В группе из семи человек 4 хорошиста и 3 отличника. Для участия в конкурсе выбрали три человека. Какова вероятность, что из них не более одного отличника?

Решение:

Не более одного отличника, т. е. один отличник или ни одного (1 или 0)

n- число всех исходов (отобрали 3 человека из 7),

m-число благоприятных исходов ( взяли 0 человек из 3 отличников и 3 человека из 4 хорошистов или 1 человека из 3отличников и 2 человека из 4 хорошистов)


Ответ:


Задания для самостоятельного решения

Практическая работа № 17

Тема: «Вычисление вероятности события»


Случайным образом выбираются числа из множества чисел . Какова вероятность того, что оно чётное?

Студент из 50 экзаменационных вопросов знает ответ на 40 вопросов. Студенту задано 3 вопроса. Какова вероятность того, что он ответит хотя бы на один вопрос?

Наугад взятый телефонный номер состоит из 5 цифр. Какова вероятность того, что в нём все цифры разные?


В лаборатории 10 новых микроскопов и 15 бывших в употреблении. Какова вероятность того, что взятый наудачу микроскоп окажется новым?


2

Из партии в 30 деталей без брака и 6 с браком берут наудачу 5 деталей. Какова вероятность того, что, по крайней мере, 4 детали без брака?

Некто, набирая номер телефона, забыл последнюю цифру. Какова вероятность того, что набирая её случайным образом, он правильно наберёт номер?

Брошены 2 игральные кости. Чему равна вероятность, что произведение выпавших очков окажется равным 5?

В ящике 60 красных шаров и 30 синих. Наудачу извлекают 5 шаров. Какова вероятность того, что из взятых шаров 3 синих?


Определить вероятность того, что номер автомашины не содержит четырёх одинаковых цифр?


Студент знает 30 вопросов из 40 вопросов программы. Какова вероятность того, что студент не знает хотя бы один ответ на вопрос из трёх предложенных?

3

На шести одинаковых карточках написаны буквы А, В, К, М, О, С. Карточки перемешивают и раскладывают наугад в ряд. Какова вероятность того, что при этом получится слово МОСКВА?

В урне 5 белых, 12 чёрных, 7 красных и 6 синих шаров. Вынимают один шар. Какова вероятность того, что этот шар цветной?


В первом ящике находятся 3 белых и 5 чёрных шаров, во втором – 1 белый и 4 чёрных. Из каждого ящика вынули по одному шару. Какова вероятность того, что все шары белые?

Из 25 лотерейных билетов 4 выигрышных. Наудачу вынимают 3 билета. Какова вероятность того, что среди них не более одного выигрышного?


Из слова «математика» наугад выбирается 1 буква. Какова вероятность того, что это будет гласная буква?


Из урны, содержащей 4 белых и 6 чёрных шаров, вынимают 3 шара. Какова вероятность того, что вынутые шары одного цвета?


4

В группе 15 студентов, из них 7 девушек. Группе нужно послать 5 человек на собрание. Какова вероятность того, что пойдут 2 юношей и 3 девушки?

Из колоды в 36 карт вынимается 2 карты. Какова вероятность того, что обе карты пиковой масти?


Наудачу взятый телефонный номер состоит из 4 цифр. Какова вероятность того, что в нём все цифры кратные 3?


Студент знает 20 вопросов из 25. Какова вероятность того, что предложенный вопрос студент не знает?


Определить вероятность того, что номер первой встретившейся автомашины не содержит трёх одинаковых цифр?


Из 25 экзаменационных билетов 5 «хороших». Студент берёт 2 билета. Какова вероятность того, что среди них один «хороший»?







Из слова «математика» наугад выбирается 1 буква. Какова вероятность того, что это будет буква «М»?


Студент знает 20 вопросов из 25. Какова вероятность того, что предложенный вопрос студент знает?


Наугад взятый телефонный номер состоит из 4 цифр. Какова вероятность того, что в нём все цифры разные?

6

Наудачу взятый телефонный номер состоит из 6 цифр. Какова вероятность того, что в нём все цифры чётные?


. Из партии, в которой 24 детали без дефекта и 6 с дефектом, берут наудачу 3 детали. Какова вероятность того, что, по крайней мере, 1 деталь без дефекта?

Из 10 узлов карданной передачи 2 получили высокую оценку ОТК. Какова вероятность того, что среди взятых наудачу 5 узлов хотя бы один высокого качества?

На отдельных карточках написаны цифры 1, 2, 3, 4, 5. Карточки раскладывают в порядке появления. Какова вероятность того, что появится число 12345?

Студент знает 25 из 30 вопросов программы. Какова вероятность того, что студент знает оба вопроса, содержащиеся в его экзаменационном билете?

Телефонный номер состоит из 6 цифр. Какова вероятность того, что в нём все цифры нечётные?


7

Студент знает 30 вопросов из 40 вопросов программы. Какова вероятность того, что студент знает хотя бы один ответ на вопрос из трёх предложенных?


Среди 30 студентов группы, из которых 12 девушек, разыгрывается 3 билета. Какова вероятность того, что среди обладателей билетов окажутся 2 юноши?

Наугад взятый телефонный номер состоит из 6 цифр. Какова вероятность того, что в нём все цифры разные?


В первом ящике 3 красных шара и 5 синих, во втором – 4 красных и 2 синих. Вынули из каждого ящика по одному шару. Какова вероятность того, что они одного цвета?

Студент из 50 экзаменационных вопросов знает ответ на 40 вопросов. Студенту задано 3 вопроса. Какова вероятность того, что он ответит по крайней мере на один вопрос

В урне 10 шаров: 4 белых и 6 чёрных. Вынули 3 шара. Какова вероятность того, что среди вынутых шаров окажутся 2 белых и 1 чёрный

8

Из колоды в 36 карт достаётся одна. Какова вероятность того, что она «красная»?


Стрелок произвёл 100 выстрелов по мишени, причём поразил мишень 45 раз. Какова вероятность того, что стрелок поразил мишень?

В книге 100 страниц. Какова вероятность того, что наугад открытая страница будет иметь порядковый номер, кратный 10?


Из колоды в 36 карт выбирают 4 карты. Какова вероятность того, что среди них окажутся 3 дамы?


Из 100 деталей, среди которых 10 бракованных, наугад берутся 5 деталей. Какова вероятность того, что все детали без брака?


В старинной игре в кости необходимо было для выигрыша получить при бросании двух игральных костей сумму очков, превосходящую 9. Найти вероятность выпадения 9 очков.

9

В один день в городе N родилось 30 детей, из них 16 мальчика. Какова вероятность того, что у «некто» родились мальчики-близнецы?


Брошены 2 игральные кости. Чему равна вероятность, что сумма выпавших очков равна 5?


Случайным образом выбираются числа из множества чисел . Какова вероятность того, что оно делится на 3?


В лаборатории 10 новых микроскопов и 15 бывших в употреблении. Какова вероятность того, что из двух, взятых наудачу микроскопов окажется новым только один?

Некто, набирая номер телефона, забыл две последних цифры. Какова вероятность того, что набирая её случайным образом, он правильно наберёт номер?

. Из партии в 20 деталей без брака и 5 с браком берут наудачу 3 деталей. Какова вероятность того, что, по крайней мере, 2 детали без брака?


10

Из трёх юношей и двух девушек выбирается комиссия из трёх человек. Какова вероятность того, что в комиссию попадут одна девушка и два юноши?

Наудачу взятый телефонный номер состоит из 4 цифр. Какова вероятность того, что в нём все цифры кратные 2?


Из колоды в 36 карт вынимается 3 карты. Какова вероятность того, что 2 карты пиковой масти?


В группе 12 студентов, среди которых 8 отличников. По списку наудачу выбирают 5 студентов. Какова вероятность того, что среди них 3 отличника?

Из слова «математика» наугад выбирается 1 буква. Какова вероятность того, что это будет буква «а»?


Определить вероятность того, что номер автомашины не содержит пяти одинаковых цифр?




Литература


  1. Башмаков М.И. Математика: учебник для учреждений нач. и сред. проф. образования /М.И. Башмаков.- 2-е изд., стер.- М.: Издательский центр «Академия», 2013.- 256 с.

  2. Башмаков М.И. Математика. Задачник: учеб. пособие для студ. учреждений сред. проф. образования /М.И. Башмаков.-5-е изд., стер.- М.: Издательский центр «Академия», 2014.- 416 с.

  3. Башмаков М.И. Математика. Сборник задач профильной направленности: учеб. пособие для учреждений нач. и сред. проф. образования / М.И. Башмаков.- М.: Издательский центр «Академия», 2012.- 208 с.



Просмотрено: 0%
Просмотрено: 0%
Скачать материал
Скачать материал "Сборник методических указаний для выполнения практических работ дисциплины "Математика: алгебра и начала математического анализа, геометрия""

Методические разработки к Вашему уроку:

Получите новую специальность за 2 месяца

Специалист в области обращения с отходами

Получите профессию

Технолог-калькулятор общественного питания

за 6 месяцев

Пройти курс

Рабочие листы
к вашим урокам

Скачать

Скачать материал

Найдите материал к любому уроку, указав свой предмет (категорию), класс, учебник и тему:

6 665 181 материал в базе

Скачать материал

Вам будут интересны эти курсы:

Оставьте свой комментарий

Авторизуйтесь, чтобы задавать вопросы.

  • Скачать материал
    • 01.09.2016 6144
    • DOCX 4.8 мбайт
    • 25 скачиваний
    • Оцените материал:
  • Настоящий материал опубликован пользователем Лабгаева Эмма Владимировна. Инфоурок является информационным посредником и предоставляет пользователям возможность размещать на сайте методические материалы. Всю ответственность за опубликованные материалы, содержащиеся в них сведения, а также за соблюдение авторских прав несут пользователи, загрузившие материал на сайт

    Если Вы считаете, что материал нарушает авторские права либо по каким-то другим причинам должен быть удален с сайта, Вы можете оставить жалобу на материал.

    Удалить материал
  • Автор материала

    Лабгаева Эмма Владимировна
    Лабгаева Эмма Владимировна
    • На сайте: 8 лет и 5 месяцев
    • Подписчики: 79
    • Всего просмотров: 58026
    • Всего материалов: 40

Ваша скидка на курсы

40%
Скидка для нового слушателя. Войдите на сайт, чтобы применить скидку к любому курсу
Курсы со скидкой

Курс профессиональной переподготовки

Бухгалтер

Бухгалтер

500/1000 ч.

Подать заявку О курсе
  • Сейчас обучается 25 человек из 18 регионов

Курс повышения квалификации

Реализация межпредметных связей при обучении математике в системе основного и среднего общего образования

36 ч. — 144 ч.

от 1700 руб. от 850 руб.
Подать заявку О курсе
  • Сейчас обучается 22 человека из 14 регионов
  • Этот курс уже прошли 94 человека

Курс профессиональной переподготовки

Математика: теория и методика преподавания в образовательной организации

Учитель математики

300/600 ч.

от 7900 руб. от 3650 руб.
Подать заявку О курсе
  • Сейчас обучается 1258 человек из 84 регионов
  • Этот курс уже прошли 3 810 человек

Курс повышения квалификации

Ментальная арифметика: умножение и деление

36 ч. — 144 ч.

от 1700 руб. от 850 руб.
Подать заявку О курсе
  • Сейчас обучается 226 человек из 54 регионов
  • Этот курс уже прошли 329 человек

Мини-курс

Инвестиционная деятельность и проектный менеджмен

3 ч.

780 руб. 390 руб.
Подать заявку О курсе

Мини-курс

Техническое обслуживание и диагностика сельскохозяйственной техники

5 ч.

780 руб. 390 руб.
Подать заявку О курсе

Мини-курс

Психология взаимоотношений, прощения и самопонимания

6 ч.

780 руб. 390 руб.
Подать заявку О курсе
  • Сейчас обучается 131 человек из 53 регионов
  • Этот курс уже прошли 48 человек